To aid in immobilizing the head of a client after thyroidectomy the nurse would obtain

  • Journal List
  • Nature Public Health Emergency Collection
  • PMC7120678

Surgery. 2014 Jul 19 : 591–699.

Guest Editor [s]: Christian de Virgilio,1 Paul N. Frank,2 and Areg Grigorian3

1Department of Surgery, Harbor-UCLA Medical Center, Torrance, California USA

2General Surgery, Harbor-UCLA Medical Center, Torrance, California USA

3Department of Surgery, University of California, Irvine, Orange, California USA

Question Sets and Answers

Acute Care Surgery

Questions

  1. A 55-year-old man presents with a mass in the left groin that is intermittently painful. The mass protrudes upon straining and reduces when he is in the supine position. With the patient standing, there is an obvious mass in his left scrotum that protrudes from the internal ring and becomes more prominent when the patient coughs. Elective surgery is recommended. At surgery, the posterior wall of the hernia sac feels very thickened and is consistent with a possible sliding hernia. Which of the following is true regarding this type of hernia?

    • [A]

      Every attempt should be made to excise the entire sac

    • [B]

      It poses a higher risk of colon injury during repair

    • [C]

      It is more common on the right side

    • [D]

      It is most often associated with direct inguinal hernias

    • [E]

      The hernia sac should be divided at the internal ring

  2. A 66-year-old woman presents to her family doctor complaining of a pain in her left groin that has appeared intermittently over the past several months. On physical exam, a soft mass is palpated in her left groin, below the inguinal ligament, and near her femoral pulse. On palpation, the mass is soft and slightly tender and disappears with gentle compression. Which of the following is true regarding these types of hernias?

    • [A]

      They are the most common hernia type in women

    • [B]

      The risk of strangulation is relatively low

    • [C]

      The hernia sac travels lateral to the femoral vein

    • [D]

      If discovered incidentally and the patient is asymptomatic, repair is not indicated

    • [E]

      It is associated with multigravida

  3. A 30-year-old woman is recovering from an open cholecystectomy in the hospital. On the second postoperative day, she begins to complain of cramping abdominal pain without vomiting. She has no past medical or surgical history, and her postoperative course has been unremarkable. She is receiving oral hydrocodone for pain and is on a clear liquid diet. She has a temperature of 99.5 °F, blood pressure is 128/84 mmHg, and pulse is 82/min. Her physical exam is significant for absent bowel sounds, a mildly distended abdomen with mild diffuse tenderness without rebound or guarding. Which of the following would most benefit her abdominal findings?

    • [A]

      Encouraging ambulation

    • [B]

      Placement of a nasogastric tube

    • [C]

      Neostigmine

    • [D]

      Conversion of hydrocodone to a nonsteroidal anti-inflammatory drug

    • [E]

      Return to the operating room for exploration

  4. A Richter’s hernia:

    • [A]

      Most often contains colon or bladder in the posterior aspect of the sac

    • [B]

      Has a low risk of incarceration

    • [C]

      Most commonly presents as a small bowel obstruction

    • [D]

      Can mislead the clinician as strangulated bowel can easily be missed

    • [E]

      Should be manually reduced in the emergency department provided there is no evidence of bowel obstruction

  5. A 55-year-old schizophrenic homeless man arrives to the ED with abdominal pain and vomiting. He reports that the abdominal pain started yesterday and has been worsening. He is afebrile, blood pressure is 122/86 mmHg, and heart rate is 116/min. In the ED he vomits green emesis without blood. His last bowel movement was 48 h ago. Physical examination reveals a large scar in his right upper quadrant. On abdominal examination, the abdomen is distended, with hyperactive bowel sounds, and is tympanic to percussion, with mild diffuse tenderness, and no rebound or guarding. WBC is 9 × 103/μL [normal 4.1–10.9 × 103/μL]. Abdominal series shows dilated loops of bowel with multiple air fluids levels. After fluid resuscitation, what is the most appropriate next step in management?

    • [A]

      Nasogastric tube suction

    • [B]

      Laparoscopy

    • [C]

      Exploratory midline laparotomy

    • [D]

      Intravenous erythromycin

    • [E]

      CT scan of the abdomen

  6. A worried mother presents to you with concerns that her 6-month-old boy has a large protrusion at his belly button that is worse when he cries but reduces when he is sleeping. On exam you palpate a 1 cm fascial defect at his umbilicus. Which of the following is true about this condition?

    • [A]

      Elective repair is recommended

    • [B]

      The condition is associated with cardiac anomalies

    • [C]

      The size of the defect predicts that it will not likely close on its own

    • [D]

      The risk of incarceration is significant

    • [E]

      Repair should be delayed until the child is 4 years old

  7. One week after open repair of a large right scrotal hernia, a 45-year-old male returns complaining of severe pain in his right testicle. On physical exam, the testicle appears to be slightly swollen and very tender to palpation. Doppler study demonstrates no flow within the right testicle with normal flow in the left. Which of the following is true about this condition?

    • [A]

      It is most commonly due to thrombosis of the pampiniform plexus

    • [B]

      Urgent exploration of the right testicle is recommended

    • [C]

      It is most likely due to transection of the testicular artery

    • [D]

      It most likely represents testicular torsion

    • [E]

      The testicle will likely remain permanently enlarged

  8. Following open inguinal hernia repair, a 50-year-old male complains of numbness and burning pain on the scrotum. This most likely represents injury to:

    • [A]

      The genital branch of the genitofemoral nerve

    • [B]

      The femoral branch of the genitofemoral nerve

    • [C]

      The ilioinguinal nerve

    • [D]

      The lateral femoral cutaneous nerve

    • [E]

      The iliohypogastric nerve

  9. A 65-year-old male presents to the ED with nausea, vomiting, and severe abdominal pain. Past history is significant for prior sigmoid colectomy for diverticulitis 10 years ago. On physical exam, his temperature is 100.9 °F, blood pressure is 110/80 mmHg, and heart rate is 110/min. His abdomen has a well-healed midline scar and is distended. Bowel sounds are hyperactive with occasional rushes and tinkles. He has marked right upper quadrant tenderness to palpation with guarding. The rest of the abdominal exam is unremarkable. Abdominal series demonstrates one loop of markedly distended small bowel in the right upper quadrant with an air fluid level. No gas is seen in the colon or rectum. Laboratory values demonstrate a WBC count of 18 × 103/μL [normal 4.1–10.9 × 103/μL] with 15 % bands and a serum lactate of 5 mmol/L [normal 0.5–1.6 mmol/L], BUN 30 mg/dL [7–21 mg/dL], and creatinine 1.2 mg/dL [0.5–1.4 mg/dL]. Amylase, lipase, and liver chemistries are normal. NG tube and IV fluids are given. What is the next step in the management?

    • [A]

      Exploratory laparotomy

    • [B]

      Admit for close observation

    • [C]

      Upper GI with small bowel follow through with barium

    • [D]

      Upper GI with small bowel follow through with Gastrografin

    • [E]

      Right upper quadrant ultrasound

Answers

1. Answer B

Sliding inguinal hernias have a much higher risk of colonic injury during repair than other hernias. This is because the posterior wall of the hernia sac is formed by a retroperitoneal organ [colon or bladder]. A clue to the presence of a sliding hernia is the finding of a thickened posterior wall of the hernia sac at surgery, in association with a large indirect hernia [D] that has descended into the scrotum [direct hernias rarely descend into the scrotum]. Attempting to completely excise the hernia sac [A] [which is otherwise normally done], or to divide the sac completely at the internal ring [E] [which is again normally recommended], would result in dividing the bowel or bladder. Sliding hernias are more common on the left side [C] [the sigmoid colon is less fixed and more likely to slide down than the right colon]. A sliding hernia is an indirect inguinal hernia [D].

2. Answer E

Multigravida causes stretching of the abdominal musculature and increases the risk of femoral hernia. Femoral hernias occur in the femoral canal, inferior to the inguinal ligament traversing the empty space medial [C] to the femoral vein [recall the mnemonic “NAVEL” {from lateral to medial: femoral nerve, artery, vein, empty space, lymphatic}]. The most common type of hernia in women, and in men, is an indirect inguinal hernia [A]. Although femoral hernias appear infrequently [10 % of all hernias], they occur more commonly in females and have the highest risk of strangulation [B]. Because of the high risk of strangulation, surgical repair of a femoral hernia is indicated [D] once diagnosed, regardless of whether the patient is having symptoms.

3. Answer D

Always consider a nonmechanical postoperative ileus in patients that have had a recent surgery. This occurs in up to 50 % of patients that have undergone abdominal surgery. Although the exact cause has not been elucidated, it most likely involves impaired peristalsis of intestinal contents. Inflammatory mediators [e.g., recent surgery] and opioid analgesics are thought to contribute to the development of postoperative ileus. Initial management should begin with changing pain medication to a non-opiate analgesic. Encouraging ambulation [A] should also be done for all postoperative patients, but is not as imperative as discontinuing opiates. If the patient’s postoperative ileus continues with worsening symptoms [e.g., emesis], bowel decompression including a NGT [B] can be considered. Returning to the OR for exploration [E] is inappropriate for postoperative ileus. Neostigmine is used in patients with pseudo-obstruction [Ogilvie’s syndrome].

4. Answer D

With a Richter’s hernia, only one wall of the bowel protrudes into the hernia sac [A]. That segment of bowel is prone to incarceration and strangulation but does so without associated symptoms, signs, or radiologic evidence of SBO [C]. Therefore, it may easily mislead clinicians into thinking that the hernia is not incarcerated [B]. Manual reduction of hernias [including Richter’s] should not be attempted if strangulation is suspected as dead bowel will be reduced into the peritoneum. Strangulation should be suspected in the presence of fever, leukocytosis, acidosis, severe pain, or marked erythema overlying the skin of the hernia. It is often difficult to palpate a Richter’s hernia, and it should be reduced in the operating room [E].

5. Answer A

This patient has evidence [on history, physical, and radiologic imaging] of a small bowel obstruction [SBO] that is most likely secondary to adhesions from prior surgery [scar in RUQ]. SBO from adhesions can present many years after surgery. The initial management of SBO includes placing the patient NPO, aggressive intravenous fluid resuscitation [the patient is tachycardic and likely very dehydrated], and NG tube placement. Aside from the salutatory effect of NG decompression on the distended bowel, patients with SBO are at risk of aspiration. Once the patient has been adequately resuscitated, CT scan [E] with oral contrast is recommended as it is useful in confirming the diagnosis of SBO, determining if the SBO is partial or complete, and ruling out other diagnosis. Most patients with SBO due to adhesions improve with these maneuvers, and do not require surgery. Operative management [C] with laparotomy and lysis of adhesions should be considered in the following conditions: if the patient demonstrates evidence of clinical deterioration as manifest by increasing pain, tenderness, fever, leukocytosis, or acidosis. Operative management can be achieved either via open laparotomy or laparoscopy [B]. Evidence of a complete SBO is a relative indication for surgery, but recent studies suggest that some of these patients resolve with nonoperative management as well. Intravenous erythromycin acts as a prokinetic agent and has some utility for gastroparesis, but not for a SBO [D].

6. Answer E

This patient has an umbilical hernia, which is a common finding in newborns. It is recommended that repair be delayed [A] until after the child is 4 years old, unless the defect is larger than 2 cm, the defect is growing, or there is evidence of strangulation. Umbilical hernias are not associated with the VACTERL [vertebral, anal, cardiac, tracheoesophageal fistula, renal, limb] complex of anomalies [B]. Defects smaller than 2 cm will likely close spontaneously [C]. It is very rare for umbilical hernias in children to incarcerate [D].

7. Answer A

This patient likely has ischemic orchitis secondary to damage to or thrombosis of the pampiniform plexus. This is most likely to occur in patients with large or densely adhesed hernia sacs. The condition is usually self-limited [E], so urgent exploration [B] is not indicated. Ischemic orchitis is more commonly caused by injury to the pampiniform plexus than to the testicular artery [C]. Testicular torsion [D] is less likely than a vascular injury in this case, although both would present with acute testicular pain and decreased or absent Doppler signal.

8. Answer A

The genital branch of the genitofemoral nerve provides sensation to the scrotum and the cremaster reflex. The femoral branch of the genitofemoral nerve [B] provides sensation to the proximal medial thigh. The ilioinguinal nerve [C] provides sensation to the lower abdomen and medial thigh. The lateral femoral cutaneous nerve [D] provides sensation to the lateral thigh as low as the knee. The iliohypogastric nerve [E] supplies the gluteal region.

9. Answer A

This patient has a SBO with evidence of ischemic or gangrenous bowel most likely secondary to adhesions from past surgery [e.g., sigmoidectomy]. Necrotic bowel generally does not occur in association with a SBO unless there is a closed-loop obstruction. A closed-loop obstruction is a particularly dangerous form of bowel obstruction in which a segment of intestine is obstructed both proximally and distally. Gas and fluid accumulate within this segment of bowel, and cannot escape. This progresses rapidly to strangulation with risk of ischemia, gangrene, and subsequent perforation. Clues to ischemic bowel include the presence of acidosis, fever, leukocytosis, and severe localized pain [unusual for SBO]. As such the patient will need exploratory laparotomy, and any bowel that is obviously nonviable needs to be resected. Most patients with SBO [without necrotic bowel] due to adhesions improve with conservative management, and do not require surgery. Observation is not appropriate for this patient [B]. Upper GI studies [C–D] would not be indicated since this patient has strong evidence of necrotic bowel and requires urgent surgical intervention. RUQ ultrasound [E] is appropriate in the workup for cholelithiasis.

Breast Cancer

Questions

  1. A 30-year-old female presents with bloody discharge from her left breast that she has noticed intermittently for the past month. She denies any palpable breast mass, weight loss, fevers, or night sweats. She has no medical history or family history of breast cancer. The skin around the breast and areola are normal with no rashes or lesions. No breast mass is palpable, and there is no axillary lymphadenopathy. Ultrasound did not reveal any masses. What is the most likely diagnosis?

    • [A]

      Fibrocystic changes

    • [B]

      Intraductal papilloma

    • [C]

      Ductal carcinoma in situ [DCIS]

    • [D]

      Paget’s disease of the breast

    • [E]

      Infiltrating ductal carcinoma

  2. A 61-year-old female presents with swelling and redness of her entire left breast that has persisted for 4 weeks. On physical exam her temperature is 98.7 °F, pulse is 82/min, blood pressure is 136/78 mmHg, and respirations are 16/min. Her left breast appears larger than her right one. The entire breast is warm, and the skin is edematous. No breast masses are palpable. There is no nipple discharge or rashes. There are several palpable enlarged lymph nodes in her left axilla. Ultrasound and mammography show thickening of the skin but otherwise no masses. Which of the following is the best option for further management?

    • [A]

      Punch biopsy of skin

    • [B]

      Oral antibiotics

    • [C]

      Intravenous antibiotics

    • [D]

      Nonsteroidal anti-inflammatory drugs

    • [E]

      Incision and drainage

  3. A 36-year-old woman is evaluated for a lump in her right breast that she noticed 5 months ago. She denies any nipple discharge, nipple retraction, or skin changes. She has no family history of breast cancer. On physical exam, the breasts appear normal. Palpation reveals a 1 cm dominant lump in the left upper quadrant that does not appear to be fixed to the surrounding structures. The patient has no other dominant masses in either breast. There is no axillary lymphadenopathy. Mammogram is negative. What is the next step in the management?

    • [A]

      Ultrasound-guided core needle biopsy

    • [B]

      Fine-needle aspiration

    • [C]

      MRI

    • [D]

      Follow-up clinical breast exam in 3 months

    • [E]

      Genetic testing

  4. A 31-year-old breastfeeding female comes to the doctor for localized swelling, redness, and pain of the left breast. She also reports muscle aches and fatigue. On physical exam her temperature is 101.1 °F, pulse is 82/min, blood pressure is 126/68 mmHg, and respirations are 16/min. Physical exam reveals a localized area of erythema and warmth in the left breast with no palpable masses. There is no axillary lymphadenopathy. What is the most likely next course of action?

    • [A]

      Biopsy

    • [B]

      Antibiotic treatment and continue breast feeding

    • [C]

      Antibiotic treatment and encourage bottle-feeding only

    • [D]

      Diagnostic mammography

    • [E]

      Incision and drainage

  5. A 17-year-old female presents with breast pain that she noticed for several months. She states that she feels multiple breast masses in both breasts. She denies any weight loss, fevers, or night sweats. She has no medical history or family history of breast cancer. Physical examination reveals that her heart has a regular rate and rhythm. The skin around the breast and areola are normal with no rashes or lesions. No solitary breast masses are palpable, but both breasts are lumpy and painful to palpation, most notably in the upper outer quadrants. There is no axillary lymphadenopathy. What is the most appropriate next step in management?

    • [A]

      Diagnostic mammography

    • [B]

      Excisional biopsy

    • [C]

      Ultrasound-guided core needle biopsy

    • [D]

      Reassurance and reexamine in 1 month

    • [E]

      Fine-needle aspiration [FNA]

  6. A 71-year-old woman is evaluated for a lump in her right breast that she noticed 3 weeks ago. She denies any nipple discharge, nipple retraction, or skin changes. She has a sister who was diagnosed with breast cancer at the age of 57. She had menarche at the age of 9 and menopause at the age of 56. She had two children, one at the age of 39 and the other at the age of 41. On physical exam, the breasts are normal on inspection. Palpation reveals a 1.5 cm dominant lump that does not appear to be fixed to the surrounding structures in the left upper, outer quadrant. The patient has no other dominant masses in either breast. There is no axillary lymphadenopathy. What is the biggest risk factor in this patient predisposing her to breast cancer?

    • [A]

      Early menarche

    • [B]

      Family history of breast cancer

    • [C]

      Older age

    • [D]

      Age at first pregnancy

    • [E]

      Late menopause

  7. A 50-year-old woman comes to clinic to discuss treatment for a new diagnosis of breast cancer. Her annual screening mammogram revealed a 1.3 cm mass in the right breast. The patient does not have any other breast masses, skin changes, nipple discharge, or axillary adenopathy. Mammography revealed no other suspicious calcifications within the breast. Biopsy of the mass was performed and revealed infiltrating ductal carcinoma. Estrogen receptor, progesterone receptor, and Her2/neu receptor testing were negative. Which of the following is the best option for the management of this patient’s breast cancer?

    • [A]

      Lumpectomy and breast irradiation

    • [B]

      Lumpectomy and hormone therapy

    • [C]

      Lumpectomy and chemotherapy

    • [D]

      Lumpectomy, sentinel node biopsy, and breast irradiation

    • [E]

      Lumpectomy, sentinel node biopsy, breast irradiation, and chemotherapy

  8. A 65-year-old woman returns to clinic for a 3-month follow-up. Three months ago she developed a pruritic, erythematous, ulcerated rash surrounding the areola of her right breast. She tried hydrocortisone 1 % on the lesion at the recommendation of her primary care physician, but the lesion persisted. She has no history of skin diseases in the family. She takes warfarin for atrial fibrillation. She started a new medication, hydrochlorothiazide, for hypertension about 3 months ago. Otherwise, she is healthy. What is the best next step in the management of this patient?

    • [A]

      Punch biopsy of the skin lesion

    • [B]

      Change hydrocortisone 1 % to triamcinolone to treat eczema

    • [C]

      Treatment with antibiotics

    • [D]

      Oral steroid course to treat psoriasis

    • [E]

      Increase the dose of hydrocortisone

  9. A 57-year-old woman comes to clinic to discuss surgical treatment for a new diagnosis of breast cancer. Her annual screening mammogram revealed a 1.7 cm mass in the right breast. Biopsy of the mass was performed and revealed infiltrating ductal carcinoma. Estrogen receptor and progesterone receptor testing were negative, while HER-2 receptor testing was positive. In addition to lumpectomy and breast irradiation, the treating doctor decides to add hormonal therapy with trastuzumab to the treating regimen. What study must be done prior to starting trastuzumab?

    • [A]

      TSH and free T4

    • [B]

      Liver function tests

    • [C]

      Echocardiogram

    • [D]

      Creatinine clearance

    • [E]

      CXR

  10. A 45-year-old female undergoes screening mammography which demonstrates an area of suspicious microscopic calcification in her right upper outer breast. Stereotactic-guided biopsy confirms ductal carcinoma in situ [DCIS]. Which of the following is true about this condition?

    • [A]

      It should be excised to a negative margin

    • [B]

      It is considered a marker for malignancy in either breast

    • [C]

      The cribriform type has a worse prognosis than the comedo type

    • [D]

      It does not occur in men

    • [E]

      Radiation therapy is an acceptable alternative to surgical excision

Answers

1. Answer B

Although bloody nipple discharge should raise concern for cancer, intraductal papilloma is the most common cause of bloody nipple discharge. This is a benign breast tumor arising from the proliferation of mammary duct epithelium that classically occurs in females 20–40 years of age. Treatment includes excision, which is diagnostic as well as curative. Fibrocystic changes [A] are a common cause of breast pain in young females. Patients report painful breast tissue before menses with improvement during menstruation. Physical exam reveals fibrotic tissue and cystic, lumpy tissue. It may be associated with bilateral serous discharge. DCIS [C] and infiltrating ductal carcinoma [E] are more common in older women. DCIS most often presents as suspicious calcifications on mammography, and not with bloody nipple discharge. Although breast cancer can present with bloody nipple discharge, it is less common than intraductal papilloma, especially in a young woman. Paget’s disease of the breast [D] causes an eczematous lesion on the breast that is associated with an underlying breast carcinoma. Given that this patient’s skin exam is normal, this diagnosis is unlikely.

2. Answer A

The patient most likely has inflammatory breast carcinoma, an especially aggressive type of breast cancer. Inflammatory breast cancer can be easily confused with mastitis, as there is usually no palpable breast mass and ultrasound and mammography similarly are often negative. As such, it is imperative to perform a biopsy of the skin, which may show cancer cells invading the subdermal lymphatics. Additional workup should include a breast MRI [which is more likely to show the breast cancer in this setting than ultrasound and mammogram], as well as consideration for needle biopsy of the lymph nodes. Antibiotics [B–C] or NSAIDs [D] would be inappropriate. Incision and drainage [E] would be appropriate if there was an indication on physical examination or evidence of a breast abscess on ultrasound. Inflammatory breast carcinoma typically presents as swelling of the breast and with edematous skin due to obstruction of subdermal lymphatics by tumor [termed peau d’orange, meaning orange peel in French]. At presentation, positive lymph node involvement is frequent, and approximately one-third of patients have distant metastases. Inflammatory breast carcinoma can present during pregnancy and should be suspected if suspected mastitis does not respond to appropriate antibiotic treatment.

3. Answer A

A diagnostic mammogram should be ordered in a woman over the age of 30 who presents with a new breast mass. Mammography helps to look for suspicious calcifications in other areas of the affected breast, characterize the mass, as well as evaluate the contralateral breast. It is important to note that the mammogram may be normal despite the presence of a palpable breast cancer. For this reason, a tissue biopsy is recommended for palpable breast masses regardless of the mammogram results. Tissue sampling is best performed via ultrasound-guided core needle biopsy. Ultrasound also provides more information about the mass [cystic vs. solid]. Fine-needle aspiration [B] is rarely used as it relies on cytology rather than histology. MRI [C] is not routinely needed. Follow-up examination in 3 months [D] without a biopsy would be inappropriate. Genetic testing [E] would be indicated if this patient had a strong family history of breast or ovarian cancer, but would not be done until tissue diagnosis of breast cancer is confirmed.

4. Answer B

The patient most likely has lactation mastitis. Lactation mastitis is a localized, painful inflammation of the breast accompanied by fever and malaise occurring in breastfeeding women. The diagnosis of mastitis is made clinically based on an erythematous, tender, swollen area of one breast associated with fever in a nursing mother. Other symptoms may include muscle pain [myalgias] and malaise. Transmission occurs via introduction of bacteria in small breaks in the skin caused by the trauma of breastfeeding. Most cases of lactation mastitis are a result of an infection by Staphylococcus aureus. Treatment consists of antibiotics to cover skin flora, symptomatic relief with analgesics including anti-inflammatory agents such as ibuprofen, and cold compresses to reduce local pain and swelling. Patients should be encouraged to continue breastfeeding [C] as this helps relieve any ductal obstruction that might be contributing to the infection. Biopsy [A] would be appropriate if the patient has suspected inflammatory breast carcinoma. Although very rare, inflammatory breast carcinoma can occur during pregnancy. If mastitis fails to resolve after antibiotics, then consideration should be given to performing a biopsy of the skin. Diagnostic mammography [D] would not be indicated at this time. Incision and drainage [E] is appropriate if there was evidence of a localized abscess with fluctuance. Ultrasound can help differentiate mastitis from a breast abscess.

5. Answer D

The history and physical exam is most consistent with a diagnosis of fibrocystic changes of the breast, which is considered a normal variant of the breast in adolescents and young adults. Patients will present with painful breast tissue before menses that improves during menstruation. On examination, fibrotic tissue may be palpated and is generally found in the upper outer quadrants of the breast. This patient should be counseled and instructed to look for these changes with a follow-up appointment in a month. Persistent cystic breast lesions can be evaluated and treated with fine-needle aspiration [E], although this is not be needed in children and adolescents. Cystic lesions that resolve with aspiration should be reevaluated with ultrasonography 3 months after aspiration [C]. Excisional biopsy [B] may be warranted for cystic lesions that do not resolve with aspiration or for suspicious solid lesions. Diagnostic mammography [A] is not indicated for adolescents and should be reserved for females >30 years old who present with a breast mass.

6. Answer C

The most important risk factors for breast cancer are female gender, increasing age, and a family history of premenopausal breast cancer. A new breast mass in a woman over the age of 50 should be considered cancer until proven otherwise, as it carries the highest relative risk of being cancer. A family history of breast cancer [B] can also significantly increase the risk of breast cancer, particularly if diagnosed in a premenopausal woman. The majority of inherited breast cancers are associated with BRCA1 or BRCA2 gene mutations. Other important risk factors associated with a slightly higher risk of developing breast cancer include early menarche [A], nulliparity or older age at first full-term pregnancy [D], and/or late menopause [E].

7. Answer E

This patient is diagnosed with infiltrating ductal carcinoma. Treatment for stage I and II breast cancers includes the option of breast conserving therapy [BCT], which consists of excision of the primary tumor [lumpectomy], sentinel lymph node biopsy [SLNB], followed by radiation therapy to the remaining breast. Studies have shown that breast conserving therapy leads to survival rates that are equivalent to that of mastectomy [though a higher local recurrence rate], while providing a more aesthetically pleasing surgical result. Triple negative breast cancers [ER, PR and Her2/neu receptor] are thought to have a worse prognosis as it is insensitive to some of the best therapies [tamoxifen and aromatase inhibitors for hormone positive, and trastuzumab for Her2/neu positive]. As such, chemotherapy is recommended postoperatively.

8. Answer A

The presentation is concerning for Paget’s disease of the breast. This presents as an eczematous, scaling, and ulcerating lesion around the areola. Paget’s disease of the breast is a type of DCIS that extends into the ducts to involve the skin of the nipple. Patients are initially misdiagnosed with a skin condition, including eczema and psoriasis, and receive a variety of ointments that do not resolve the lesion. Paget’s disease of the breast is almost always associated with an underlying carcinoma and must be diagnosed via biopsy of the lesion. Trying different regimens of steroids and antibiotics is inappropriate given the high likelihood that she has cancer [B–E].

9. Answer C

Trastuzumab is a monoclonal antibody that blocks the HER-2 receptors. The medication is used in the treatment of HER-2-positive breast cancers to help reduce recurrence and improves survival. Since there is a high risk of cardiomyopathy in patients receiving trastuzumab, it is recommended that all patients receive an echocardiogram prior to initiating therapy with trastuzumab. An alternative is to obtain a MUGA scan [multigated acquisition scan], which is a nuclear study that evaluates ventricular function. Trastuzumab-related cardiotoxicity is most often manifested by an asymptomatic decrease in ejection fraction. The optimal surveillance for trastuzumab-related cardiotoxicity is not well defined. The remaining answer choices are not needed prior to starting trastuzumab [A–B, D–E].

10. Answer A

DCIS is characterized by malignant epithelial cells within the mammary ductal system, without invasion into the surrounding stroma. Comedo-type DCIS is typically high grade and associated with a worse prognosis [C]. DCIS lesions have a high risk of subsequent invasive carcinoma at the site of the DCIS. As such if left unresected, it will often progress to invasive ductal cancer. Thus the mainstay of DCIS treatment is lumpectomy [excision of entire lesion with negative margins]. Lobular carcinoma in situ is considered a marker for malignancy in either breast [B]. Breast cancer in males is rare [1 % of all breast cancers] with most cases identified as invasive ductal carcinoma. DCIS can occur in men but is even more rare, as DCIS most often presents as abnormal calcifications on mammogram [D]. Radiation therapy can be used in combination with surgical excision, but cannot replace it [E].

Cardiothoracic

Questions

  1. A 65-year-old male presents with a painful nodule in his wrist that is determined to be a ganglion cyst. Despite attempts at aspiration, it recurs. He is unable to work as a computer programmer, is on disability, and is feeling depressed. He is scheduled for wrist surgery. He reports having been discharged 1 week ago for an episode of chest pain. Troponins were elevated at that time, but there was no elevation of his ST segment. Which of the following is the best recommendation?

    • [A]

      Proceed with surgery with intraoperative transesophageal echocardiography

    • [B]

      Proceed with surgery but perform under local anesthesia with sedation

    • [C]

      Proceed with surgery only if echocardiogram shows normal ejection fraction

    • [D]

      Proceed with surgery after aggressive beta blockade to get heart rate into low 60s

    • [E]

      Postpone surgery for at least 4 weeks

  2. A 65-year-old male is about to undergo an elective inguinal hernia repair. Which of the following findings on history or physical would portend the highest operative risk?

    • [A]

      Systolic, crescendo-decrescendo murmur at the sternal border of the right second intercostal space radiating into neck

    • [B]

      A history of myocardial infarction 10 years ago

    • [C]

      Insulin-dependent diabetes mellitus with an elevated HgbA1C

    • [D]

      Renal insufficiency not yet on dialysis

    • [E]

      Smoking

  3. A 65-year-old male undergoes a videoscopic right upper lobectomy for squamous cell lung cancer. On postoperative day one, he suddenly develops chest pain and diaphoresis. Blood pressure is 120/60 mmHg, and heart rate is 80/min. Serial highly sensitive troponin I assays demonstrate levels of 0.4, 0.3, and 0.01 ng/dl. ECG demonstrates nonspecific T wave changes with no ST segment elevation. Following the administration of oxygen, morphine, aspirin, and a beta-blocker, his symptoms resolve. What is the next step in the management?

    • [A]

      Intravenous thrombolytic therapy

    • [B]

      Percutaneous coronary intervention without stenting

    • [C]

      Percutaneous coronary intervention with stenting

    • [D]

      Coronary artery bypass graft [CABG]

    • [E]

      Continue medical management and reevaluate as outpatient in 4–6 weeks

  4. A 17-year-old African American male presents for a pre-participation physical before track season. A harsh systolic murmur is heard at the second right intercostal space. He denies ever experiencing chest pain, dizziness, or difficulty breathing. Which of the following would be expected on further workup?

    • [A]

      T wave inversion on ECG

    • [B]

      Laterally displaced PMI on palpation

    • [C]

      Weak femoral pulses compared to brachial pulses

    • [D]

      Increased intensity of the murmur with Valsalva maneuver

    • [E]

      Increased intensity of the murmur with squatting

  5. A 65-year-old woman arrives to the ED complaining of chest pain. Her past medical history includes hypertension, atherosclerosis, and coronary artery disease. She underwent a coronary artery bypass graft [CABG] 3 weeks ago for three-vessel disease. She reports that her chest pain worsens with inspiration and lessens when leaning forward. A friction rub is heard on auscultation. ECG shows global ST elevation. What is the most likely diagnosis?

    • [A]

      Myocarditis

    • [B]

      Myocardial infarction

    • [C]

      Cardiac tamponade

    • [D]

      Acute pericarditis

    • [E]

      Pulmonary embolism

  6. An obese 52-year-old man with a 50-pack-year smoking history and hypertension controlled with chlorthalidone presents to a remote hospital without interventional capabilities with 30 min of crushing chest pain radiating to his left arm and jaw. Troponin and CK-MB levels are elevated, and ECG shows ST segment elevations in leads V1 through V4. He is treated with thrombolytic therapy, and his symptoms resolve. The next morning, the patient is found dead in his bed. Which of the following is the most likely cause of death?

    • [A]

      Ventricular free wall rupture

    • [B]

      Embolic stroke

    • [C]

      Ventricular arrhythmia

    • [D]

      Post-MI pericarditis

    • [E]

      Overwhelming infection

  7. A 65-year-old female has breast cancer and a remote history of congestive heart failure. Her physician is planning to administer a chemotherapeutic agent that has potential for cardiac toxicity. Which of the following is the most accurate test to measure ejection fraction?

    • [A]

      Multi Gated Acquisition Scan [MUGA] scan

    • [B]

      Echocardiography

    • [C]

      Electrocardiogram

    • [D]

      Coronary angiography

    • [E]

      Exercise stress test

  8. A 76-year-old man is driven to the ED by his wife and is complaining of severe chest pain that started 30 min ago. He denies abdominal or extremity pain. Pulses in arms and legs are 2+. His kidney function is normal. CT scan shows an aortic dissection. Which of the following findings on CT scan would most strongly indicate the need for urgent surgery?

    • [A]

      Dissection of entire descending thoracic aorta

    • [B]

      Involvement of common iliac arteries

    • [C]

      Involvement of renal arteries

    • [D]

      Extension into mesenteric vessels

    • [E]

      Involvement of origin of innominate artery

  9. A 65-year-old female is diagnosed with aortic dissection beginning 2 cm distal to the left subclavian artery and extending distally. Her blood pressure is 180/70 mmHg, and her heart rate is 88/min. Peripheral pulses are all 2+, and her abdomen is soft and non-tender. What is the next best step in treatment?

    • [A]

      Surgical repair

    • [B]

      Aggressive IV fluids

    • [C]

      Labetalol drip

    • [D]

      Endovascular repair

    • [E]

      Nicardipine drip

  10. A 65-year-old man is rushed to the ED by ambulance after he suddenly lost strength and sensation in his left leg and arm. He was hospitalized 2 months ago with a NSTEMI. He is compliant with all of his medications and had been recovering well until the present episode. ECG shows normal sinus rhythm without evidence of ischemia. Chest X-ray is unremarkable. Carotid ultrasounds show < 30 % stenosis bilaterally. What is the most likely etiology of the patient’s present symptoms?

    • [A]

      Ventricular thromboembolism

    • [B]

      Septic embolism to the brain

    • [C]

      Type A dissection involving the right carotid artery

    • [D]

      Thromboembolism from the left atrial appendage

    • [E]

      Paradoxical venous thromboembolism

  11. A 66-year-old man is recovering in the ICU after receiving a CABG for coronary artery disease. On the fourth postoperative day, he complains of chest pain. He is sweating, anxious, short of breath, and nauseated. ECG shows evidence of right-sided MI. His blood pressure is 98/65 mmHg. What is the next best step in management?

    • [A]

      Administer 1 L of normal saline

    • [B]

      Nitroglycerin

    • [C]

      Nitroprusside

    • [D]

      Nifedipine

    • [E]

      Lisinopril

  12. A 63-year-old woman with diabetes is recovering in the ICU after receiving a CABG for coronary artery disease. On the sixth postoperative day, she starts complaining of chest pain. Her temperature is 101.4 °F, blood pressure is 108/72 mmHg, and pulse is 125/min. On physical exam, there is drainage from her sternal wound, and there is a crunching sound heard with a stethoscope over the precordium during systole. The sternum feels somewhat unstable to palpation. Her laboratory examination is significant for an elevated white blood count [16.7 × 103/μL]. What is the most likely diagnosis?

    • [A]

      Acute pericarditis

    • [B]

      Postoperative MI

    • [C]

      Empyema

    • [D]

      Acute mediastinitis

    • [E]

      Pneumonia

  13. A 75-year-old male with severe aortic stenosis has a routine check-up at his primary care doctor. Which of the following symptoms portends the worst prognosis?

    • [A]

      Exertional chest pain

    • [B]

      Swollen legs

    • [C]

      Fainting spells

    • [D]

      Mid-systolic murmur heard loudest at the upper right sternal border

    • [E]

      Small head nodding movements at each heartbeat

  14. Which of the following is the most important risk factor for aortic dissection?

    • [A]

      History of coronary artery bypass grafting [CABG]

    • [B]

      Giant cell arteritis

    • [C]

      Pregnancy

    • [D]

      Hypertension

    • [E]

      Bicuspid aortic valve

  15. A patient is diagnosed with type A aortic dissection, and there is concern for cardiac tamponade. Which of the following findings would be the MOST consistent with cardiac tamponade?

    • [A]

      Pulsus bisferiens

    • [B]

      Watson’s water hammer pulse

    • [C]

      Peaked T waves

    • [D]

      Equalization of central pressures

    • [E]

      Pulsus alternans

  16. A 67-year-old male is diagnosed with a type B aortic dissection. At the time of initial presentation on the previous day, his blood pressure was 178/110 mmHg. He was treated with intravenous beta-blocker, and his blood pressure was reduced to 112/60 mmHg and has remained in that range. However, one day later, he suddenly develops severe abdominal pain. His blood pressure is measured to be 110/56 mmHg. Which of the following is the most likely explanation?

    • [A]

      C. difficile infection

    • [B]

      Occlusion of the superior mesenteric artery

    • [C]

      Pancreatitis

    • [D]

      Aortoenteric fistula

    • [E]

      Diverticulitis

  17. A 40-year-old male presents with acute chest pain and nausea. Serum troponin levels are elevated, and the ECG demonstrates ST segment elevation. Which of the following would be the strongest contraindication to intravenous thrombolytic therapy?

    • [A]

      Right knee arthroscopic surgery 1 month ago

    • [B]

      Recently completed antibiotic course for H. pylori infection

    • [C]

      Wide mediastinum on CXR

    • [D]

      History of alcohol abuse

    • [E]

      Endovascular aortic aneurysm repair 1 month ago

Answers

1. Answer E

Proceeding with elective surgery 1 week after an acute MI is inappropriate [A–D]. Patients with a recent MI are at significantly increased cardiac risk during noncardiac surgery, particularly within the first month after MI. Since the proposed operation is elective, options A–D would place the patient under unnecessary risk. Although performing the operation under local anesthesia with sedation [B] seems appealing, there is still considerable stress and cardiac risk with such an approach. The best recommendation for this patient is to postpone surgery for at least 4 weeks. At that point, consideration should still be given to cardiac stress testing prior to surgery or even further surgical delay, as the cardiac risk persists for at least 6 months after an MI.

2. Answer A

Major predictors of adverse postoperative cardiac events must be identified prior to elective noncardiac surgery. These include recent [within 1 month] MI, unstable or severe angina, decompensated CHF, and significant arrhythmias. Such cardiac conditions require postponing surgery and performing further cardiac workup. A systolic, crescendo-decrescendo murmur at the sternal border of the right second intercostal space radiating into the neck is highly suggestive of aortic stenosis and would require an echocardiogram to rule out severe aortic stenosis. Aortic stenosis impairs coronary perfusion, which can become further exacerbated during induction of anesthesia. From all the choices listed, it portends the highest operative risk. Lee’s revised cardiac risk index identifies intermediate risk factors; these include known coronary artery disease [B] history of CHF, history of stroke or TIA, insulin-dependent diabetes [C], creatinine > 2.0 mg/dl [possibly D], and high-risk surgery [i.e., aortic]. Adding a point for each factor and a assigning a score [from 0 to 6] are highly effective in stratifying cardiac risk. Interestingly, smoking [E] has not been shown to be an independent risk factor for adverse perioperative cardiac events in most studies.

3. Answer E

The patient has suffered a postoperative NSTEMI. Most NSTEMI [as opposed to a STEMI] in the postoperative setting are managed without percutaneous coronary intervention [PCI] with a combination of oxygen, morphine for pain relief, aspirin, and a beta-blocker. Optimally, an additional antiplatelet agent [such as clopidogrel] and intravenous heparin are also given, but this depends on how recent the operation was and the potential for postoperative bleeding. Consideration should be given to stress testing at 4–6 weeks after surgery, and depending on the results, PCI is then considered. Urgent PCI [B,C] is indicated in the setting of a STEMI, and in certain high-risk NSTEMIs [continued rise in troponins, ongoing chest pain], but will require clopidogrel [again may not be desirable so soon after surgery] if a stent is placed. The patient described has a down trend of troponins and relief of symptoms, further supporting medical management. Emergent CABG [D] would be considered if PCI fails or is not technically feasible with severe three-vessel disease. Emergent operations for acute MI continue to have a high mortality despite many technological advances in myocardial protection. Thrombolytic therapy [A] is an alternative when PCI is not available but would be contraindicated within 2–3 weeks of major surgery.

4. Answer D

The patient likely has hypertrophic obstructive cardiomyopathy, an asymmetric thickening of the ventricular septum that creates a narrowing of the left ventricular outflow tract. Vigorous exercise places him at increased risk of sudden cardiac death. T wave inversion [A] would be found in ischemic heart disease, very unlikely in an otherwise healthy 17-year-old. Laterally displaced PMI [B] would be found in patients with congestive heart failure, also very unlikely in this patient. Weak femoral pulses compared to brachial pulses [C] is a finding in coarctation of the aorta, and would not create the characteristic murmur. Murmurs due to aortic regurgitation, mitral regurgitation, and ventricular septal defect [VSD] increase in intensity with squatting [E].

5. Answer D

Acute pericarditis is inflammation in the pericardial sac accompanied by pericardial effusion. It can occur following post-MI [termed Dressler’s syndrome], chest radiation, or recent heart surgery. Patients present with pleuritic chest pain that lessens when leaning forward, friction rub heard on auscultation, global ST elevation, and PR depression. Patients with myocarditis [A] usually present with signs and symptoms of acute decompensating heart failure [e.g., tachycardia, gallop, mitral regurgitation, and edema]. Chest pain accompanied with MI [C] would not be expected to lessen with leaning forward. Furthermore, global ST elevation would not be expected. Cardiac tamponade [C] can occur once the effusion reaches a critical mass in which cardiac output is compromised. Pulmonary embolism [E] can present with pleuritic chest pain, but it will not be influenced by positioning and is more likely to have ECG findings suggestive of right heart failure.

6. Answer C

It is important to know the timing of causes of death after MI. In the first 48 h after MI, death is likely due to ventricular arrhythmia. If arrhythmia occurs after 48 h, an implantable defibrillator should be placed. Ruptures of the myocardium, either as a ventricular septal rupture or free wall rupture [A], usually do not occur until 4–5 days after MI, at which point the dead myocardium has been weakened by the body’s inflammatory response. Post-MI pericarditis, also known as Dressler’s syndrome, [D] usually occurs weeks or months after MI or cardiac surgery. An embolic stroke [B] would present with sudden onset of numbness on one side of the body, cranial nerve deficits, and/or aphasia. It is unlikely to cause death so quickly. There is no reason to believe the patient has sustained an overwhelming infection [E].

7. Answer A

The MUGA scan is the most accurate test in measuring ejection fraction. It is a noninvasive nuclear test that uses a radioactive isotope called technetium to evaluate the function of the ventricles. Though not as accurate, an echocardiogram [B] is used more commonly because it is cheaper and more readily available and can look for valve function as well as focal areas of wall motion abnormality. Electrocardiogram [C] and exercise stress test are unable to measure a patient’s ejection fraction. Coronary angiography [D] is considered the gold standard in identifying coronary artery disease and can estimate ejection fraction, but is not as accurate.

8. Answer E

It is important to rapidly identify Stanford type A dissections, as they require urgent surgical intervention due to the fact that they can lead to cardiac tamponade, acute aortic valve insufficiency, acute MI, and stroke. A Stanford A dissection involves the ascending aorta and/or the aortic arch. Thus an aortic dissection involving the innominate artery is a Stanford type A. Stanford type B aortic dissection is more common. A Stanford type B dissection begins in the descending aorta, distal to the takeoff of the left subclavian artery [A–D]. Stanford Type B dissections are much less likely to cause acute complications since the ascending aorta/aortic arch are not involved. A type B dissection may involve the mesenteric, renal, or iliac arteries, but not occlude them, as blood may continue to flow normally [either though the true or the false lumen]. Most can be managed medically with blood pressure control [beta-blockers]. Surgical intervention is needed if the involvement of these vessels leads to malperfusion [such as leg ischemia, bowel ischemia, or renal failure].

9. Answer C

Based on the description of the site of the dissection, this is a type B aortic dissection. These are usually managed medically [A] unless the patient has evidence of malperfusion. Since her peripheral pulses are all 2+ and her abdomen is soft and non-tender, there is no evidence of malperfusion. The goal is to maintain a relatively low blood pressure in order to minimize stress on the aorta. Aggressive IV fluids [B] will not reduce blood pressure and may actually raise it. Nicardipine [E] will lower blood pressure, but intravenous beta-blocker is the treatment of choice because it also reduces the rate of pressure increase with each beat of the heart, which lowers the stress on the aortic wall. Endovascular therapy [D] is not routinely needed for most type B dissections.

10. Answer A

Patients with a recent history of myocardial infarction are at risk of thrombus formation on the scarred endocardium, which can then embolize to the brain and cause a stroke. Patients with a recent history of MI and evidence of thrombus on echocardiography should be treated with warfarin to maintain an INR of 2–3 and followed up within 3 months. Thromboembolism from the left atrial appendage [D] is a concern in patients with atrial fibrillation. Paradoxical venous thromboembolism [E] is a concern in patients with an atrial septal defect or patent foramen ovale, wherein a deep venous thrombus can travel through the defect into the left heart and ultimately to the brain. Septic embolism [B] is a concern in IV drug abusers and can lead to cerebral abscess. Type A dissection [C] would usually present with severe chest pain radiating to the back.

11. Answer A

This patient has a postoperative right-sided MI, resulting in compromised cardiac output secondary to decreased preload. One of the steps in management of right-sided MI is to administer fluids to help increase filling of the heart. Avoid nitrates [B, C] in these patients as it may further reduce preload. Acutely, patients with MI need oxygen, aspirin, analgesics, and beta-blockers. Dihydropyridine calcium channel blockers, such as nifedipine [D], are contraindicated in MI because of the associated peripheral vasodilation that may lead to reactive tachycardia and subsequently result in even more stress on the heart. ACE inhibitors [E] should be considered for long-term treatment after the acute episode has resolved.

12. Answer D

This patient’s presentation is most concerning for acute mediastinitis. This is a life-threatening infection of the mediastinum with a very high mortality rate that is most commonly associated with cardiac surgery. The incidence rate is 1–2 % following CABG. The source of infection may be a sternal wound infection, combined with instability of the sternum that permits bacteria to enter the mediastinum. Hamman’s sign is a crunching sound heard with a stethoscope over the precordium during systole and is suggestive of acute mediastinitis. Patients will frequently present with chest pain, increased drainage from sternal wound, fevers, and leukocytosis. Chest radiograph findings include pneumomediastinum and/or air-fluid levels within the mediastinum. A CT scan can also support the diagnosis by demonstrating dehiscence of the sternum and stranding, fluid and air pockets within the anterior mediastinum. Management includes surgical debridement, drainage, antibiotics, and rewiring the sternum. Acute pericarditis [A] will present with pleuritic chest pain that lessens when leaning forward, friction rub heard on auscultation, and characteristic ECG findings [global ST elevation]. Pneumonia [E] would present with shortness of breath, productive cough, and abnormal lung sounds. Postoperative MI [B] would not be expected to present with evidence of systemic inflammation. Empyema [C] is defined as pus in the pleural space, and would not explain the physical exam findings of sternal instability and Hamman’s sign. CT scan would demonstrate a loculated fluid collection within the right or left pleural cavity.

13. Answer B

The classic signs of severe aortic stenosis are angina [A], syncope [C], and congestive heart failure [which may manifest as swollen legs]. Of the three, congestive heart failure portends the worst prognosis, with median survival as low as 2 years. A loud mid-systolic murmur [D] indicates hemodynamically significant obstruction but is a better prognostic sign than an absent murmur, which indicates low blood flow across the valve. Small head nodding movements with each heartbeat [E] are known as de Musset’s sign and is found in aortic regurgitation.

14. Answer D

All of the above are risk factors for aortic dissection [A–C, E]. However, the most significant risk factor for aortic dissection is systemic hypertension.

15. Answer D

In cardiac tamponade, fluid [blood or effusion] in the pericardial space externally compresses the heart, which limits diastolic filling and reduces stroke volume. Since pericardial fluid is free flowing, the pressure is distributed equally along the pericardium. As this continues the rising pressure in the pericardium is transmitted to all four cardiac chambers resulting in equalization of central pressures. Pulsus bisferiens [A], also known as a biphasic pulse, refers to two strong systolic pulses with a mid-systolic dip, in other words, two pulses during systole. It can be seen in aortic regurgitation with or without aortic stenosis and hypertrophic cardiomyopathy. Watson’s water hammer pulse [B] is a pulse with a rapid upstroke and descent seen in patients with aortic regurgitation. Peaked T waves [C] is most often associated with hyperkalemia. It is unlikely to be seen in patients with cardiac tamponade since their ECG findings are characteristically low voltage. Pulsus alternans [E] is a physical exam finding wherein the amplitude of a peripheral pulse changes from beat to beat associated with changing systolic blood pressure. It is most commonly caused by left ventricular failure.

16. Answer B

Sudden onset of severe abdominal pain in association with an aortic dissection should always raise suspicion for malperfusion of the bowel which can lead to bowel gangrene and death. This most likely would occur if the dissection extends into, and suddenly occludes, the superior mesenteric artery, which supplies blood to the bowel from the ligament of Treitz to the mid-transverse colon. It is also important to recognize that bowel ischemia early on causes excruciating pain in the absence of peritonitis [“pain out of proportion to physical exam”]. He has not been on broad-spectrum antibiotics, and has no reason to have C. difficile infection [A], which most often presents with vague abdominal pain and diarrhea. Pancreatitis [C] presents with epigastric pain radiating to the back, nausea, vomiting, anorexia, fever, and tachycardia and is most commonly associated with cholelithiasis and alcohol abuse. Aortoenteric fistula [D] is a possible long-term sequela in patients who have had an intra-aortic synthetic graft placed. Diverticulitis [E] is a common cause of left lower quadrant abdominal pain in elderly patients, and does not typically cause such sudden severe pain.

17. Answer C

Wide mediastinum on chest X-ray is concerning for aortic dissection. Patients with type A aortic dissection can present with coronary artery malperfusion and thus have a similar presentation as an acute MI. Suspected aortic dissection is considered an absolute contraindication to thrombolysis in patients with myocardial infarction. The remaining choices [A–B, D–E] are all relative contraindications for intravenous thrombolytics.

Endocrine

Questions

  1. A 27-year-old woman has 3 months of intermittent spells of severe headache, heart palpitations, and sweating. A pregnancy test at her primary care doctor’s office is positive. Further workup reveals that her plasma metanephrine level is 220 pg/ml [normal 12–60 pg/ml]. What is the next step in establishing the diagnosis?

    • [A]

      CT abdomen

    • [B]

      Repeat plasma metanephrine level after the patient has delivered

    • [C]

      MRI abdomen

    • [D]

      I131-MIBG scan

    • [E]

      Reassure patient that symptoms are related to pregnancy

  2. Preoperative medical optimization for a patient with a pheochromocytoma routinely includes:

    • [A]

      Octreotide drip for 24 h before surgery

    • [B]

      Control of hypertension with beta-blockade as first-line agent

    • [C]

      Control of hypertension with alpha-blockade as first-line agent

    • [D]

      Metyrosine

    • [E]

      Diuretics for blood pressure management

  3. A 55-year-old otherwise healthy patient undergoes a non-contrast CT abdomen to evaluate for possible kidney stones and is incidentally noted to have a 8 cm mass in the left adrenal gland. The mass has irregular borders and high attenuation, suggesting a lipid-poor lesion, and appears to be adherent to the kidney. How should this patient be managed?

    • [A]

      Observation with repeat CT scan in 3 months

    • [B]

      Open adrenalectomy

    • [C]

      Laparoscopic adrenalectomy

    • [D]

      Radiation therapy

    • [E]

      Percutaneous biopsy

  4. A 50-year-old female has been recently diagnosed with primary hyperparathyroidism. She comes in to her doctor complaining of increased bone pain in her legs. She is found to have elevated serum calcium, alkaline phosphate, and PTH. Her doctor decides to order plain films of her lower extremities. The radiographs show very thin bones with a stress fracture and bowing of both femur bones. She also has characteristic cysts with a moth-eaten appearance. What is the most likely diagnosis?

    • [A]

      Osteoporosis

    • [B]

      Osteopetrosis

    • [C]

      Osteomalacia

    • [D]

      Osteitis fibrosa cystica

    • [E]

      Paget’s disease of the bone

  5. A 60-year-old man is found to have a 3 cm right adrenal mass on CT scan which was obtained a month earlier following a MVC. He is asymptomatic, and does not report a history of hypertension or diabetes. What is the most appropriate next step in management?

    • [A]

      Repeat CT scan in 6 months

    • [B]

      Percutaneous needle biopsy

    • [C]

      Biochemical workup for hormone excess

    • [D]

      Laparoscopic adrenalectomy

    • [E]

      No further follow-up is necessary

  6. An elderly nursing home patient has been bedridden for several months due to a series of debilitating strokes. Past medical history is significant for hypertension, controlled with a diuretic, and Paget’s disease. Recently, the patient has been complaining of vague abdominal pain, constipation, and depressed mood. On physical examination, the patient is alert and oriented. Abdominal examination is unremarkable. Which of the following electrolyte abnormalities would most likely explanation her symptoms?

    • [A]

      Hyponatremia

    • [B]

      Hypernatremia

    • [C]

      Hyperphosphatemia

    • [D]

      Hypocalcemia

    • [E]

      Hypercalcemia

  7. Which of the following is most consistent with an aldosterone-secreting adrenal adenoma?

    • [A]

      Hyperglycemia, hirsutism, and abdominal striae

    • [B]

      Hypertension and hyperkalemia

    • [C]

      Hypertension and hypokalemia

    • [D]

      Elevated plasma metanephrine and hypertension

    • [E]

      Increased vanillylmandelic acid excretion and hypertension

  8. A 35-year-old patient presents for a follow-up visit for an elevated serum calcium level of 12.8 mg/dL and an elevated PTH. He is a thin man without a significant past medical history. He reports that for the past 2 weeks he has been experiencing loose stools, polydipsia, and polyuria. On physical exam he was found to have large erythematous erosions with blisters over the lower abdomen. Which tumor would best explain the patient’s symptoms and rash?

    • [A]

      Insulinoma

    • [B]

      Prolactinoma

    • [C]

      VIPoma

    • [D]

      Glucagonoma

    • [E]

      Adrenal adenoma

  9. A 32-year-old female patient arrives for follow-up for new-onset hypertension. She was started on hydrochlorothiazide 6 months ago. During her visit, she was found to have a blood pressure of 152/98 mmHg. She also complains of recent episodes where she experiences sudden palpitations, chest pain, diaphoresis, headache, and anxiety. Her laboratory exam demonstrates a calcium of 13.2 mg/dl [normal 8.5–10.2 mg/dl], PTH of 102 pg/ml [10–55 pg/ml], and an elevated plasma metanephrine. Which of the following would be an important additional component in the workup?

    • [A]

      Fasting blood glucose

    • [B]

      Prolactin level

    • [C]

      MRI of the sella turcica

    • [D]

      Serum calcitonin

    • [E]

      Serum gastrin level

  10. A 45-year-old man has had hazy vision for the past month, particularly when he is driving at night. He also endorses small rubberlike nodules on the skin of his trunk, back, arms, and legs that are not painful and do not itch. After seeing his ophthalmologist, he is diagnosed with bilateral cataracts and is scheduled to receive elective cataract surgery. During induction of anesthesia, following intubation, the patient’s pressure increases from 110/70 to 200/90 mmHg. PaCO2 is normal as is his pH. His temperature is 101.5 °F. An esmolol drip is immediately instituted, after which BP increases to 220/90 mmHg and an ECG shows T wave inversion. What is the most likely underlying etiology?

    • [A]

      Intra-abdominal tumor

    • [B]

      Malignant hyperthermia

    • [C]

      Thyrotoxicosis

    • [D]

      Inadequate anesthetic agent

    • [E]

      Undiagnosed pituitary tumor

  11. A 12-year-old boy presents to the doctor for a lump in his neck. He is healthy with no previous medical problems. On physical examination, he has a well-defined anterior neck mass, located in the midline and above the cricoid cartilage. The mother states that she has noted the lesion since he was about 2 years old. It does not bother him. On physical examination, the mass elevates with swallowing and is non-tender. He has no cervical adenopathy and no other complaints. The neck mass is described as a hypoechoic mass on ultrasonography. A subsequent thyroid scintogram is performed and confirms the thyroid gland is in its correct anatomic position. Which of the following would be recommended next for this mass?

    • [A]

      FNA biopsy

    • [B]

      Proceed to surgical excision

    • [C]

      Reassurance and observation

    • [D]

      TSH and free T4

    • [E]

      CT scan

  12. In addition to elevated plasma free metanephrine, a change in what other serum marker can help support the diagnosis of pheochromocytoma?

    • [A]

      Plasma chromogranin A

    • [B]

      Plasma superoxide dismutase

    • [C]

      Malondialdehyde

    • [D]

      CA 19–9

    • [E]

      5-Hydroxyindoleacetic acid [HIAA]

  13. A 42-year-old man with a family history of endocrine tumors is diagnosed with MEN-2A after presenting with uncontrolled hypertension and subsequent genetic workup. He was found to have a right adrenal pheochromocytoma and asymptomatic hyperparathyroidism. What is the recommended surgical management for this patient?

    • [A]

      Parathyroid surgery first, followed by adrenalectomy

    • [B]

      Adrenalectomy first, followed by parathyroid surgery

    • [C]

      Medical conditioning for 2 weeks prior to adrenalectomy, followed by parathyroid surgery

    • [D]

      Medical conditioning for 2 weeks prior to simultaneous parathyroid surgery and adrenalectomy

    • [E]

      Medical conditioning for 2 weeks followed by adrenalectomy only

  14. A 39-year-old man is recovering from bilateral adrenalectomy for a pheochromocytoma. On his second postoperative day, he begins to complain of nausea, vomiting, weakness, blurry vision, and mild abdominal pain. His temperature is 102.9 °F, and blood pressure is 90/68 mmHg. His ECG shows sinus tachycardia. His laboratory examination from that morning showed:

    • Sodium: 134 mEq/L [137–145 mEq/L]

    • Potassium: 5.8 mEq/L [3.6–5.0 mEq/L]

    • Calcium: 7.4 mg/dL [8.9–10.4 mg/dL]

    • BUN: 12 mg/dL [7–21 mg/dL]

    • Creatinine: 1.2 mg/dL [0.5–1.4 mg/dL]

    • Glucose: 70 mg/dL [65–110 mg/dL]

    • Albumin: 2.4 g/dL [3.5–4.8 g/dL]

    • WBC 10.5 × 103/μL [4.1–10.9 × 103/μL]

    Which of the following can best explain this patient’s current presentation?

    • [A]

      Volume depletion

    • [B]

      Sepsis

    • [C]

      Hypocalcemia

    • [D]

      Low cortisol

    • [E]

      Loss of catecholamine production

  15. A 56-year-old woman is recovering after undergoing total thyroidectomy for papillary carcinoma. Her temperature is 99.8 °F, blood pressure is 120/80 mmHg, and pulse is 90/min. During her postoperative examination by the intern, the patient complains of numbness and tingling around her mouth and in her hands and feet. What could have been done postoperatively to anticipate and potentially remedy these symptoms?

    • [A]

      Check magnesium

    • [B]

      Check parathyroid hormone

    • [C]

      Check potassium

    • [D]

      Check TSH and free T4

    • [E]

      Carotid ultrasound

  16. A 42-year-old man presents with new-onset hypertension and elevated hemoglobin [19 mg/dL] and hematocrit [58 %] levels on subsequent laboratory examination. A CT scan demonstrates bilateral adrenal masses suspicious for pheochromocytoma. His elevated hemoglobin and hematocrit are believed to be secondary to a paraneoplastic syndrome. What other tumor is classically associated with this same paraneoplastic syndrome?

    • [A]

      Glioblastoma multiforme

    • [B]

      Hemangioblastoma

    • [C]

      Colorectal cancer

    • [D]

      Wilms’ tumor

    • [E]

      Osteosarcoma

  17. Which of the following is true regarding paragangliomas [extra-adrenal pheochromocytomas]?

    • [A]

      The most common location is within the kidney

    • [B]

      There is a decreased association with familial syndromes [e.g., MEN-2, Von Hippel–Lindau] compared to pheochromocytomas

    • [C]

      They are less likely to be malignant compared to pheochromocytomas

    • [D]

      Functional imaging [MIBG] is particularly useful to diagnose metastatic disease, particularly when CT/MRI are negative

    • [E]

      They are different on a cellular level from intra-adrenal pheochromocytomas

  18. A malignant pheochromocytoma is diagnosed by:

    • [A]

      Pathologic identification of high mitotic rate, cellular atypia, and capsular invasion

    • [B]

      Positive MIBG scan

    • [C]

      Presence of metastasis at sites normally devoid of chromaffin tissue

    • [D]

      Biomolecular markers

    • [E]

      The presence of intractable hypertension

  19. A 45-year-old female presents with a 2 cm painless mass in her right anterior neck that has been present for 3 months and slowly enlarging. On physical exam, the mass feels firm and moves up and down with swallowing. She denies weight loss, weight gain, heat intolerance, or anxiety. A serum TSH level is normal. The most important step in the workup is:

    • [A]

      CT scan of the neck

    • [B]

      MRI of the neck

    • [C]

      Fine-needle aspiration [FNA]

    • [D]

      Open biopsy

    • [E]

      Nuclear scan

  20. Three hours after total thyroidectomy for thyroid cancer, the patient complains of difficulty breathing. On physical examination, the patient has stridor and appears to be in moderate respiratory distress. Examination of the wound demonstrates tense swelling. The next step in the management is:

    • [A]

      Immediately reopen wound at the bedside

    • [B]

      Intubation

    • [C]

      Emergent return to the operating room for wound exploration

    • [D]

      Check oxygen saturation

    • [E]

      Send arterial blood gas

  21. During the course of a total thyroidectomy in a 40-year-old female, the surgeon divides the superior thyroid artery and vein in one large ligature. After dividing the vascular pedicle, the surgeon notices that it appears that a nerve was transected. The surgeon postoperatively should warn the patient that she will most likely have:

    • [A]

      Permanent hoarseness

    • [B]

      A droop in the corner of her mouth

    • [C]

      Difficulty swallowing

    • [D]

      Trouble hitting high notes when singing

    • [E]

      A need for a permanent tracheostomy

  22. A 45-year-old female presents to her physician complaining of abdominal pain. She has a history of recurrent kidney stones and was recently discharged from the hospital after undergoing ureteroscopic laser lithotripsy. Her laboratory examination is significant for calcium of 13.6 mg/dL [normal 8.5–10.2 mg/dL] and PTH of 112 pg/mL [10–55 pg/mL]. She is scheduled for operative management of her underlying condition. At surgery, all four parathyroid glands are identified. Only one appears to be abnormally enlarged and is removed. Confirmation of curative resection is best achieved via:

    • [A]

      Intraoperative ultrasound

    • [B]

      Intraoperative frozen section

    • [C]

      Intraoperative PTH levels

    • [D]

      Immediate postoperative serum calcium level

    • [E]

      Postoperative sestamibi scan

  23. A 35-year-old female presents with bone pain, abdominal pain, and depressed mood. Her laboratory examination is significant for calcium of 11.3 mg/dL [normal 8.5–10.2 mg/dL] and PTH of 109 pg/ml [10–55 pg/mL]. Localization of the enlarged gland or glands is best achieved by:

    • [A]

      Preoperative MRI

    • [B]

      Preoperative ultrasound

    • [C]

      Preoperative sestamibi scan

    • [D]

      Preoperative FNA

    • [E]

      Intraoperative exploration of all four glands

  24. A 38-year-old female arrives for her yearly physical. She has no complaints but was incidentally found to have laboratory markers suggestive of primary hyperparathyroidism. Subsequent workup reveals involvement of all four parathyroid glands. She remains asymptomatic. What is the best recommendation for management of this patient?

    • [A]

      Observation

    • [B]

      Surgical removal of all four glands

    • [C]

      Surgical removal of 3.5 glands

    • [D]

      Biochemical monitoring of serum calcium and serum creatinine annually

    • [E]

      Cinacalcet

Answers

1. Answer C

This patient presents with the rare but classic presentation of pheochromocytoma during pregnancy. The preferred imaging modality in pregnancy is an MRI, due to the risks of exposing the fetus to radiation with other types of imaging [A, D]. In men and non-pregnant women, CT with contrast can also be considered a first line imaging study. Pheochromocytoma is usually hyperintense on T2-weighted images due to its high water content. Failing to work up and treat a potential pheochromocytoma in pregnancy exposes the fetus and mother to a very high risk of morality during the pregnancy and delivery [B, E].

2. Answer C

Patients with pheochromocytoma are volume depleted due to intense alpha-mediated vasoconstriction. Hypertension is controlled with alpha-blockade [e.g., phenoxybenzamine] for 10–14 days before surgery. This allows for volume expansion, and the patient is encouraged to liberally intake salt and fluids. The dose is titrated until hypertensive episodes are controlled, often resulting in mild orthostatic hypotension. Beta-blockers [B] can be used to decrease reflex tachycardia once appropriate alpha-blockade has been established. Initiating beta-blocker therapy prematurely can precipitate a hypertensive crisis due to unopposed alpha-adrenergic vasoconstriction. Octreotide [A] is a somatostatin analogue that may have minimal efficacy in the palliation of symptoms from malignant pheochromocytoma, but it has no role in preparing a patient for surgery. Metyrosine [D] inhibits catecholamine production and is a secondary agent for pheochromocytoma, though now rarely used. Diuresis [E] would be contraindicated as these patients are volume depleted.

3. Answer B

This patient was incidentally found to have an adrenal mass. Guidelines for surgical resection include tumors > 6 cm, features on CT suspicious for malignancy [high attenuation, irregular borders, inhomogeneous], and those that are hormonally active. Most adrenal carcinomas are hormonally active. Thus the patient described has several indications for adrenalectomy. Open adrenalectomy is preferred when malignancy is suspected, as this allows for a wider resection with en bloc resection if adjacent structures are involved and eliminates the possibility of seeding the port sites that may occur with laparoscopic adrenalectomy [C]. Laparoscopic adrenalectomy is preferred for benign lesions. Radiation therapy [D] is not the mainstay of treatment for adrenal cortical carcinoma. Percutaneous biopsy [E] is not recommended as there are no histologic features that diagnose adrenal cortical carcinoma and a biopsy may risk seeding the biopsy tract.

4. Answer D

Osteitis fibrosa cystica is a skeletal disorder that results from a surplus of parathyroid hormone. Patients experience increased bone pain, bone fractures, and skeletal deformities with bowing of the bones. Radiographs show thin bones, fractures, and cysts with a moth-eaten appearance. Osteoporosis [A] usually occurs in elderly patients and is characterized by decreased bone density with normal mineralization. It does not have any associated cyst-like features. Similarly, osteopetrosis [B] would not have any cysts seen on plain films. Paget’s disease [E] results from overactive osteoclasts and osteoblasts leading to excessive bone turnover and is characterized by tibial bowing, kyphosis, increased cranial diameter, and deafness. Patients with Paget’s disease and osteoporosis have normal serum calcium, while patients with osteomalacia [C] would be expected to have decreased serum calcium.

5. Answer C

The first step in the evaluation of an incidentally discovered adrenal mass is to perform a biochemical workup to determine if the tumor is functional or nonfunctional [E]. In practice, it is common to order a single battery of tests: serum aldosterone, plasma renin activity, and a 24-h urine collection to simultaneously measure catecholamines, metanephrines, and cortisol. Given that this patient is normotensive, the suspicion for pheochromocytoma and hyperaldosteronism is low. In addition, adrenal masses < 6 cm are unlikely to be malignant. If the mass is found to be a hormonally active adrenal adenoma, then laparoscopic adrenalectomy [D] would be recommended. If biochemical testing reveals a nonfunctioning mass, this small lesion may be observed with interval CT scanning [A]. Percutaneous needle biopsy [B] cannot readily distinguish between benign and malignant primary adrenal tumors.

6. Answer E

Hypercalcemia can cause abdominal pain, constipation, mental status changes, and depressed mood [stones, bones, moans and groans]. Prolonged immobilization is a known cause of hypercalcemia and is seen in adolescents and in other patients with increased bone turnover such as Paget’s disease. Certain diuretics [thiazide] also cause hypercalcemia by increasing renal calcium resorption.

7. Answer C

Patients with hyperaldosteronism have hypertension and hypokalemia – not hyperkalemia [B]. Aldosterone acts on the kidney to increase sodium reabsorption, and potassium is excreted to balance the positively charged sodium ions. Hyperglycemia, hirsutism, and abdominal striae [A] are more consistent with Cushing’s syndrome. Elevated plasma metanephrine, hypertension, and increased vanillylmandelic acid excretion [D, E] are all consistent with pheochromocytoma.

8. Answer D

Elevated serum calcium combined with elevated PTH is consistent with primary hyperparathyroidism. Rarely, it can be associated with MEN-1 which includes parathyroid, pituitary, and pancreatic pathology [3Ps]. Pancreatic tumors include gastrinoma, insulinoma, VIPoma, and glucagonoma. Glucagonoma should be suspected in a patient with new-onset diabetes mellitus [even if thin], diarrhea, and the classic rash: annular, erythematous erosions with blisters over the lower abdomen [necrolytic migratory erythema]. The patient’s symptoms of polyuria and polydipsia are highly suggestive of diabetes mellitus. Insulinoma [A] is characterized by hypoglycemia, headache, visual changes, confusion, weakness, and diaphoresis. Prolactinomas [B] are excess prolactin-producing anterior pituitary tumors that may result in amenorrhea, galactorrhea, decreased libido, and gynecomastia. A VIPoma [C] [also called WDHA syndrome: watery diarrhea hypokalemia achlorhydria] presents with profuse diarrhea, but will not have any skin manifestations of the disease. An adrenal adenoma [E] is oftentimes benign, nonfunctional, and incidentally found on imaging [incidentalomas].

9. Answer D

Severe hypertension in a young patient should raise suspicion for surgically correctable causes such as aldosteronoma, Cushing’s disease, coarctation of the aorta, fibromuscular dysplasia of the renal arteries, and pheochromocytoma. Her symptoms, combined with an elevated plasma metanephrine level, make pheochromocytoma the most likely cause. The addition of labs consistent with primary hyperparathyroidism [elevated calcium and PTH] suggests she has MEN-2A which is characterized by primary hyperparathyroidism, pheochromocytoma, and medullary thyroid cancer. Calcitonin is a reliable tumor marker for medullary thyroid cancer and should always be ordered to rule out this very aggressive cancer in this patient population. Fasting blood glucose [A] [insulinoma], prolactin levels [prolactinoma] [B], MRI of the sella turcica [C] [pituitary adenoma], and serum gastrin level [E] [gastrinoma] are all associated with MEN-1.

10. Answer A

A sudden rise in blood pressure after anesthetic induction raises concern for an undiagnosed pheochromocytoma, malignant hyperthermia, and thyrotoxicosis [thyroid storm]. For each of these situations, cessation of anesthesia is recommended. There are several clues that point to pheochromocytoma as the cause. The administration of beta-blockers without alpha-blockade first leads to worsening hypertension due to unopposed alpha-mediated vasoconstriction as in the case above. Pheochromocytoma is associated with neurofibromatosis-1 which may present with skin neurofibromas [rubberlike discolored skin lesions] and cataracts. Malignant hyperthermia [B] presents with muscle rigidity [most often the masseter], a rapid increase in core body temperature, a rise in end tidal CO2, arrhythmia, and a mixed metabolic and respiratory acidosis at anesthetic induction. Treatment is immediate cessation of surgery and dantrolene. Thyrotoxicosis [C] presents in a similar fashion to malignant hyperthermia [fever, hypertension, tachycardia]; however, it is not associated with muscle rigidity or rising end tidal CO2. The associated hypertension and tachycardia respond to the administration of beta-blockade. It is due to a hypermetabolic state caused by excess thyroid hormone. Inadequate anesthetic agents [D] may lead to hypertension and tachycardia, but would not lead to high fevers. An undiagnosed pituitary tumor resulting in excess ACTH production can cause hypertension, but this will be accompanied with symptoms consistent with Cushing’s disease [e.g., truncal obesity, abdominal striae, muscle wasting, hirsutism].

11. Answer B

This patient has a thyroglossal duct cyst, which is the most common midline congenital malformation of the neck. Though present at birth, these do not often appear until age 2 as baby fat recedes. During embryological development, the thyroid originates at the base of the tongue and travels down the thyroglossal duct to the anterior neck, where it normally involutes. However, if a persistent duct remains, it may undergo cystic dilation later in life and present as a well-defined anterior neck mass, located midline and above the cricoid cartilage. Unlike a brachial cleft cyst, this elevates with tongue protrusion or swallowing. Ectopic thyroid gland may be associated with thyroglossal duct cysts so it’s necessary to confirm the thyroid gland is in its correct anatomic location prior to surgical intervention. The definitive management involves thyroglossal duct cyst excision or the Sistrunk procedure. Reassurance and observation [C] are inappropriate as thyroglossal duct cysts have a high rate of recurrent infections and a small risk of progressing to malignancy. FNA biopsy [A] is appropriate for a thyroid nodule, but not for suspected thyroglossal duct cyst. He does not have symptoms suggestive of hyper- or hypothyroidism so a thyroid panel would not be indicated [D]. CT scan [E] is unnecessary for the diagnosis, and additionally should not be performed in such a young patient secondary to significant radiation exposure.

12. Answer A

Plasma free metanephrine is highly sensitive for pheochromocytoma but is more prone to false-positive results. Plasma chromogranin A is released from neuroendocrine cells and is elevated in the majority of patients with pheochromocytoma. It is nonspecific [i.e., it is elevated in other neuroendocrine tumors] but can help confirm the diagnosis. Superoxide dismutase and malondialdehyde [B, C] are both markers for oxidative stress, and neither has been shown to be associated with pheochromocytoma. CA 19–9 [D] may be elevated in some patients with pancreatic cancer. Increased level of 5-hydroxyindoleacetic acid [HIAA] [E] would be expected in a patient with carcinoid syndrome.

13. Answer C

Patients with MEN-2A can develop pheochromocytoma, hyperparathyroidism, and medullary thyroid cancer. The definitive management for pheochromocytoma consists of medical conditioning with alpha-blockade and sometimes beta-blockade for at least 2 weeks, followed by an adrenalectomy [B]. This should be performed first [A, D–E] because a pheochromocytoma can increase the risk of complications during the surgical management of other endocrine tumors. Although he is asymptomatic with respect to his hyperparathyroidism, parathyroid surgery is generally recommended for most patients with inherited forms, as it tends to be more aggressive and presents at a much younger age. Age less than 50 is an indication for parathyroid surgery for sporadic forms as well, as the patient is more likely to suffer one of the sequelae of hyperparathyroidism.

14. Answer D

If a patient that has undergone bilateral adrenalectomy presents postoperatively with severe hypotension and hypoglycemia, suspect Addisonian crisis [acute adrenal insufficiency] and check a cortisol level. This is considered to be a life-threatening condition caused by insufficient levels of cortisol, which is responsible for maintaining blood pressure and glucose homeostasis. Patients will present with nausea, vomiting, weakness, blurry vision, and mild abdominal pain. Laboratory exam would be expected to show hypoglycemia, hyperkalemia, and mild hyponatremia. Plasma ACTH levels will be low, and a Cortrosyn [synthetic ACTH] stimulation test will demonstrate a low cortisol response. This patient should receive immediate fluid resuscitation [normal saline] and intravenous corticosteroids. Acute adrenal insufficiency does not respond to vasopressors. Additionally, it can mimic sepsis. However, he does not meet SIRS criteria. Similarly, sepsis [B] is unlikely to present with this patient’s lab abnormalities. Patients that have had major surgery should always be monitored for signs of internal hemorrhaging. Although his serum calcium is shown to be low [C], this should be corrected for hypoalbuminemia. His corrected serum calcium is 8.7 mg/dL, is within the normal range, and would not explain the hypotension [B]. Although he may be volume depleted [A], this would not cause hypoglycemia or hyperkalemia. Loss of catecholamine production [E] may accompany Addisonian crisis and is also seen after removing a pheochromocytoma. It is associated with hypotension and hypoglycemia; however, it will not cause hyperkalemia and hyponatremia.

15. Answer B

The patient most likely has hypocalcemia. Temporary hypoparathyroidism occurs in up to 30 % of patients after total thyroidectomy and generally lasts a few weeks. It is thought to be related to temporary ischemia to the adjacent parathyroid glands. Patients will complain of numbness and tingling in their hands and feet, as well as around the mouth. These patients should be managed with prompt oral calcium supplementation. Oral calcitriol may be added to increase calcium absorption from the gut. Some centers routinely check the postoperative PTH level for the purposes of anticipating hypocalcemia. Left untreated, hypocalcemic symptoms may progress to muscle twitching [including Chvostek’s sign] and ultimately tetany, which is an emergency. IV calcium [gluconate or chloride] may be given in these circumstances, but its use can generally be avoided when patients are carefully monitored postoperatively. Symptoms of hypomagnesemia [A] are indistinguishable from hypocalcemia; however, low magnesium levels are not associated with thyroidectomy. Disturbances in potassium [C] and thyroid hormone [D] would not cause the symptoms described. Carotid ultrasound [E] would be indicated if the patient developed symptoms of a stroke or transient ischemic attack [one-sided arm and leg weakness/numbness].

16. Answer B

This patient most likely has an ectopic production of erythropoietin leading to high levels of hemoglobin and hematocrit. This paraneoplastic syndrome, termed polycythemia vera, is classically associated with pheochromocytoma, renal cell carcinoma, hepatocellular carcinoma, and hemangioblastoma [A, D–E].

17. Answer D

Paragangliomas arise from extra-adrenal chromaffin tissue, with the most common location being in the abdomen [organ of Zuckerkandl]. They are essentially identical on a cellular level to intra-adrenal pheochromocytomas. However, they are more likely to have a hereditary basis [30–50% of cases] and to be malignant [15–35%]. The diagnosis is made by biochemical analysis followed by imaging localization. It is particularly important to consider a whole body functional scan due to the higher propensity for multifocal and metastatic disease.

18. Answer C

There is currently no way to establish the diagnosis of malignancy in pheochromocytoma based on histopathologic evaluation [A]. However, there are tumor characteristics that are associated with higher risk [e.g., larger size, extra-adrenal location, certain genetic mutations, and a high tumor proliferative index]. Malignancy is determined by the development of metastatic disease, defined by a recurrence in an area that normally does not have any chromaffin tissue [lymph nodes or a distant site such as the liver or lungs]. MIBG scanning [B] can be useful to identify metastatic disease, but positivity of the primary tumor on MIBG does not determine whether it is malignant. Biomolecular markers [D] can differentiate a functional tumor from nonfunctional, but is unable to rule out malignancy. Similarly, intractable hypertension [E] is not a characteristic of malignancy.

19. Answer C

The most important step in the diagnostic workup of a thyroid nodule is to obtain a tissue sample. This is best obtained via fine-needle aspiration and is best done under ultrasound guidance. Thyroid nodules greater than 1 cm in size, nodules with ultrasound characteristics suggestive of malignancy [internal microcalcifications, e.g.], or those with a history of growth should undergo ultrasound guided FNA. CT [A] or MRI [B] would be appropriate for patients found to have clinical or sonographic evidence of locally advanced thyroid cancer that may extend into the aerodigestive tract or substernal region. Open biopsy [D], done by removing an entire thyroid lobe, should be done next if FNA results are suspicious for a follicular neoplasm. Nuclear scanning [E] has a very limited role in the preoperative setting. It is more beneficial in the postoperative setting to look for recurrent or metastatic malignancy.

20. Answer A

Don’t forget the ABCs. This patient has a compromised airway and is in moderate respiratory distress. Normally, the first step to ensure an airway is via endotracheal intubation [B]. However, a neck hematoma is in a closed space that leads to compression of the airway that may render safe intubation difficult or impossible. As such, the first step is to immediately open the neck wound at the bedside to decompress the hematoma. This will typically relieve the airway obstruction. The patient can then be transported emergently to the operating room for intubation, wound exploration, adequate hemostasis, and subsequent wound closure [C]. Although thyroidectomy is considered a safe procedure, one well-known complication is airway obstruction following bleeding and hematoma formation which occurs within the first 24 h after thyroidectomy. Checking oxygen saturation [D] or waiting for labs [E] is never appropriate for a patient with a compromised airway.

21. Answer D

The superior laryngeal nerve lies adjacent to the superior thyroid artery and is thus at high risk of being injured during mobilization of the thyroid, particularly the superior pole. The external branch of the superior laryngeal nerve permits singing in a high pitch. This nerve may be injured in up to 25 % of cases but is usually asymptomatic unless the patient is a singer or voice professional. Damage to the recurrent laryngeal nerve on one side results in a paralyzed vocal cord in a median or paramedian position. This manifests as hoarseness [A] and sometimes aspiration. The rate of permanent unilateral recurrent laryngeal nerve injury during thyroidectomy should be less than 2 % in expert hands. If both recurrent laryngeal nerves were injured during a total thyroidectomy, then both vocal cords could be paralyzed, and this may lead to a compromised airway which may necessitate a permanent tracheostomy [E]. A droop in the corner of the mouth results from injury to the marginal mandibular branch of the facial nerve. Swallowing is controlled by multiple nerves [C] including the glossopharyngeal, vagus, and/or hypoglossal nerves.

22. Answer C

The surgical treatment of hyperparathyroidism depends on whether the pathology is a single adenoma [most common, remove single gland], more than one adenoma [remove abnormal ones], or four gland hyperplasia [remove 3.5 glands]. Distinguishing these entities is not always obvious. Because of the short half-life of PTH [about 4 min], intraoperative rapid PTH testing aids in determining the completeness of parathyroid resection. The most commonly used protocol involves drawing PTH levels at the time of gland excision and again 10 min post-excision. A fall of >50 % in the PTH level is associated with a 98 % long-term cure rate. Given the small size of the parathyroid glands, it is generally not recommended to biopsy all of them for frozen section [B], as such a biopsy may render all the glands ischemic. Transient hypocalcemia is expected following parathyroidectomy so postoperative serum calcium level [D] is not indicative of cure. Oral calcium supplementation can help alleviate minor symptoms. Intraoperative ultrasound [A] is sometimes used when the abnormally enlarged gland cannot be found. Sestamibi [E] may be used if recurrent or persistent hyperparathyroidism develops, but is not routinely used for confirmation of successful surgery.

23. Answer C

Sestamibi scanning involves using a radioisotope, technetium-99 m, which is taken up by cells with high mitochondrial activity. It is more accurate for single adenomas than for four gland hyperplasia. Sestamibi scanning and to a lesser extent ultrasound [B] are the most frequently used imaging tests to localize the involved gland[s] in primary hyperparathyroidism. Localizing studies are generally not indicated in secondary or tertiary hyperparathyroidism, since multiple-gland hyperplasia is the expected underlying pathology. Preoperative FNA [D] is not helpful in the workup of primary hyperparathyroidism. In about 85 % of patients, imaging will localize the abnormal parathyroid gland, and a great majority will have a single parathyroid adenoma. If localizing scans are negative, yet the diagnosis of primary hyperparathyroidism is clearly established, surgery is still performed at which time intraoperative exploration of all four glands [E] is performed.

24. Answer C

With the increasing use of routine laboratory testing, most patients with primary hyperparathyroidism are currently discovered incidentally in asymptomatic patients. Although the patients may be asymptomatic, long-standing hyperparathyroidism can lead to kidney injury and osteoporosis. Evidence of such should be sought out via bone mineral density testing as well as calculation of creatinine clearance. For patients with asymptomatic hyperparathyroidism diagnosed through laboratory screening, a 2008 consensus statement recommended the following indications for surgery:

  1. Serum calcium 1.0 mg/dL greater than the upper limit of normal

  2. Creatinine clearance reduced to direct hyperbilirubinemia

  3. [E]

    Elevation of transaminases out of proportion to alkaline phosphatase

  4. Which of the following is a risk factor for pancreatic cancer?

    • [A]

      Alcohol

    • [B]

      Smoking

    • [C]

      Prostate cancer in the family

    • [D]

      Malabsorption

    • [E]

      Pancreatic enzyme supplementation

  5. A 60-year-old woman arrives to the emergency department with bloody emesis. She has a past medical history significant for hypertension and an episode of severe pancreatitis due to alcohol abuse 1 year ago and has since developed chronic pancreatitis. Her temperature is 98.6 °F, blood pressure 110/88 mmHg, and pulse of 88/min. Esophagogastroduodenoscopy shows bleeding coming from isolated gastric varices. Which of the following is most likely to successfully treat the bleeding?

    • [A]

      Liver transplantation

    • [B]

      Endoscopic banding of the varices

    • [C]

      Endoscopic sclerotherapy

    • [D]

      TIPS [transjugular portosystemic shunt]

    • [E]

      Splenectomy

  6. A 56-year-old male undergoes a Whipple procedure for pancreatic adenocarcinoma. Two days later, there is about 30 cm3 of white/opal opaque drainage emanating from the patient’s drain. What is the most appropriate next step?

    • [A]

      Obtain abdominal CT scan

    • [B]

      Obtain abdominal ultrasound

    • [C]

      Send fluid for amylase level

    • [D]

      Start octreotide

    • [E]

      Initiate total parenteral nutrition [TPN]

  7. Which of the following is an appropriate use of CA 19–9?

    • [A]

      Screening normal, healthy patients for pancreatic cancer

    • [B]

      Screening at-risk patients for pancreatic cancer

    • [C]

      Confirming diagnosis of pancreatic cancer in patients with periampullary mass on CT

    • [D]

      Monitor for progression of disease following resection and/or adjuvant therapy

    • [E]

      None of the above

  8. A 45-year-old presents with a 1 day history of RUQ pain and tenderness, nausea, and vomiting. Physical examination is significant for marked RUQ tenderness and guarding. Laboratory values are significant for a WBC of 12 × 103/μL [normal 4.1–10.9 × 103/μL] with 10 % bands, total bilirubin of 1.2 mg/dL [0.1–1.2 mg/dL], AST of 110 μ/L [normal 5–35 μ/L], ALT of 120 μ/L [7–56 μ/L], and alkaline phosphatase of 90 μ/L [33–131 μ/L]. RUQ ultrasound reveals several gallstones, a thickened gallbladder wall, and a normal common bile duct. Optimal management consists of:

    • [A]

      Schedule for elective outpatient laparoscopic cholecystectomy

    • [B]

      Admit, IV antibiotics, laparoscopic cholecystectomy within 48 h of admission

    • [C]

      Admit, IV antibiotics for 4–5 days followed by laparoscopic cholecystectomy

    • [D]

      Admit, IV antibiotics until WBC normalizes, followed by outpatient laparoscopic cholecystectomy

    • [E]

      Admit, IV antibiotics, ERCP, followed by laparoscopic cholecystectomy

  9. A 40-year-old female presents with moderate epigastric abdominal pain. She has a history of intermittent RUQ pain after eating fatty foods. On physical examination she is afebrile with a heart rate of 100/min and blood pressure of 110/70 mmHg. She has moderate epigastric tenderness to palpation. Laboratory values are significant for a WBC of 11 × 103/μL [normal 4.1–10.9 × 103/μL] with 3 % bands, total bilirubin of 1.2 mg/dL [0.1–1.2 mg/dL], AST of 250 μ/L [5–35 μ/L], ALT of 300 μ/L [7–56 μ/L], alkaline phosphatase of 150 μ/L [33–131 μ/L], amylase of 1,300 μ/L [30–110 μ/L], and lipase of 1,100 μ/L [7–60 μ/L]. RUQ ultrasound shows numerous small gallstones, normal gallbladder wall, and a normal common bile duct diameter of 0.4 mm. On the second hospital day, her pain has resolved; she is afebrile and has a normal heart rate, and her WBC count has normalized. The amylase has decreased to 350 μ/L. Optimal management consists of:

    • [A]

      Proceed with laparoscopic cholecystectomy [LC] with intraoperative cholangiogram [IOC]

    • [B]

      Wait 4–5 more days until amylase has completely normalized, and then proceed to LC with IOC

    • [C]

      ERCP followed by LC during same hospitalization

    • [D]

      Schedule for elective outpatient LC with IOC

    • [E]

      ERCP only

  10. A 50-year-old diabetic male presents with severe RUQ pain and fevers. On physical examination, his temperature is 103.5 °F, BP is 100/60 mmHg, and heart rate is 120/min. He has severe tenderness to palpation in the RUQ. WBC is 20 × 103/μL [normal 4.1–10.9 × 103/μL] with 10 % bands, total bilirubin is 1.0 mg/dL [0.1–1.2 mg/dL], amylase is 90 μ/L [30–110 μ/L], alkaline phosphatase is 90 μ/L [33–131 μ/L], AST is 110 μ/L [normal 5–35 μ/L], and ALT is 140 μ/L [7–56 μ/L]. RUQ ultrasound shows gallstones, a normal common bile duct diameter, and a few gas bubbles within the wall of the gallbladder. IV fluids and antibiotics are administered. The next step in the management consists of:

    • [A]

      Immediate cholecystectomy

    • [B]

      Admit to ICU for 24–48 h of IV antibiotics and careful monitoring

    • [C]

      Cholecystostomy

    • [D]

      CT scan of abdomen

    • [E]

      ERCP

  11. A 58-year-old female underwent a CT scan of her abdomen and pelvis following a motor vehicle collision 1 month ago. She has no intra-abdominal injuries and was discharged from the ED. However, her gallbladder was incidentally noted to be heavily calcified. She is otherwise in good health and denies any abdominal pain. Her past history is significant for mild hypertension. Which of the following is the best recommendation?

    • [A]

      Reassure patient that no follow-up is needed

    • [B]

      Repeat CT scan in 1 year

    • [C]

      Laparoscopic cholecystectomy

    • [D]

      Check serum calcium and PTH levels

    • [E]

      Obtain ERCP with brushings

Answers

1. Answer B

Acalculous cholecystitis is a condition seen in patients that are critically ill such as those with multiorgan trauma, burns, or recent major surgery. The exact mechanism is unclear, but it is thought to be secondary to a combination of biliary stasis [from being NPO] and gallbladder ischemia as a result of hypovolemic and/or septic shock. The diagnosis can be difficult for several reasons. Patients are critically ill so a history may be unobtainable and physical exam may be unreliable. The imaging test of choice is ultrasound [US]. Findings suggestive of acalculous cholecystitis include gallbladder wall thickening and pericholecystic fluid; however, such findings are not consistent. If US is not definitive, HIDA scan is the next test and is considered positive if the gallbladder is not visualized. However, false positives are seen in patients who have been NPO for a prolonged period [which many of these critically ill patients have]. Gallstones are not implicated in this condition, and will not be seen on ultrasonography. Treatment of acalculous cholecystitis includes broad-spectrum antibiotics followed by urgent percutaneous cholecystostomy [if the patient is critically ill] or cholecystectomy [laparoscopic vs. open cholecystectomy] if the patient is stable enough to undergo general anesthesia. Acute pancreatitis [A] is in the differential; however, the patient’s lipase is normal [more specific than amylase], and the amylase is only mildly elevated [acute pancreatitis requires an elevation 3x above normal]. Mild hyperamylasemia can be seen with many intra-abdominal conditions including cholecystitis or bowel ischemia. Burn victims are at risk for stress-related mucosal damage [Curling ulcer] secondary to an inability to maintain the integrity of the gastrointestinal mucosal barrier. This may subsequently lead to perforated viscus [D] which will present with an acute abdomen and a plain film demonstrating free air under the diaphragm. Cholangiohepatitis [C] is associated with biliary parasites such as Clonorchis sinensis and is characterized by brown pigment stones that result from biliary sludge and dead bacterial cell bodies. Acute cholangitis [E] would present with evidence of cholestasis [jaundice and/or elevated liver enzymes] and biliary obstruction [dilated bile ducts on ultrasound].

2. Answer B

The most likely diagnosis is acute pancreatitis secondary to gallstones. More than half of all cases of pancreatitis are associated with either gallstones or alcohol. Patients with gallstone pancreatitis have extremely high serum amylase [sometimes in the thousands] and ALT [greater than 3x the upper limit of normal] as compared to other etiologies. A biliary etiology of pancreatitis is further supported by the elevated bilirubin and alkaline phosphatase which suggest at least a temporary obstruction of the common bile duct by a gallstone. Most gallstones only transiently obstruct the common bile duct and pass on their own. Pancreatitis due to alcohol is seen in patients with long-standing heavy alcohol abuse [which is not suggested by the history in this patient], and not following a onetime binge. Chronic pancreatitis is rare with gallstones. It is most often seen in association with long-standing alcohol abuse. Patients present with chronic epigastric pain, steatorrhea, and/or diabetes. Amylase and lipase levels are often not elevated.

3. Answer D

The patient is presenting with painless jaundice, which should be considered as due to malignancy until proven otherwise. Courvoisier’s sign is a term used to describe a palpable non-tender gallbladder that markedly distends as a result of a gradual common bile duct obstruction, most often by a pancreatic adenocarcinoma at the head of the pancreas [distal common bile duct and ampullary cancer are also in the differential]. Gallstones typically cause sudden obstruction of the biliary tree and often harbor bacteria. They are almost always associated with pain and often trigger an inflammatory response. Charcot’s triad [B] [RUQ pain, fever, and jaundice] and Reynold’s pentad [C] [the triad plus hypotension and altered mental status] are associated with acute cholangitis most often secondary to a gallstone obstructing the distal common bile duct. Murphy’s sign [E] [RUQ tenderness on palpation that stops inspiration] is associated with acute cholecystitis due to a gallstone obstructing the cystic duct. Cullen’s sign [A] is a blue-red discoloration at the umbilicus, and the appearance is a result of digested blood products in the retroperitoneum, forming methemalbumin, that then travel towards the anterior abdominal wall. It is associated with retroperitoneal bleeding, as seen with hemorrhagic pancreatitis.

4. Answer D

The presentation is most consistent with pancreatitis secondary to alcohol. Lipase is more specific for pancreatitis than amylase. The vast majority of acute pancreatitis cases are due to peripancreatic inflammation, not infection. Antibiotics [A] are therefore not beneficial. CT scan [B] is not necessary to establish the diagnosis and should be reserved for situations where the diagnosis is in question or if the patient clinically deteriorates during the subsequent hospitalization. The most appropriate management is to make the patient NPO, aggressively hydrate, and administer analgesics. Routine use of an NG tube is unnecessary. ERCP [C] is utilized in gallstone pancreatitis, specifically if the patient demonstrates evidence of concomitant cholangitis or obstructive jaundice. Librium [E] may be utilized for alcohol withdrawal, but should not be prioritized ahead of aggressive hydration. The majority of patients resolve the episode of pancreatitis within 3–5 days using conservative management.

5. Answer D

Gallstones are commonly discovered incidentally in asymptomatic patients after imaging studies that are performed for unrelated reasons. Only 2–3% of asymptomatic patients develop biliary colic per year, and only a small fraction of those patients progress to complications from gallstones [such as acute cholecystitis, pancreatitis, acute cholangitis]. For this reason, the vast majority of asymptomatic gallstones should not receive prophylactic cholecystectomy [A]. Prophylactic cholecystectomy might be considered in patients who are planning extended travel to areas without healthcare access [Antarctica in the winter]. Because patients with diabetes are at greater risk of developing complications from gallstones, some authors have recommended prophylactic cholecystectomy in diabetics. However, even in the diabetic population, it is reasonable to wait to see if symptoms develop. Ursodeoxycholic acid [B] can dissolve gallstones. However, it is completely successful in only about 1/3 of cases, only for cholesterol stones, is associated with side effects [diarrhea], and is costly. In addition, the stones may recur once the medication is stopped. ERCP [C] is an invasive procedure utilized for choledocholithiasis and acute cholangitis. Extracorporeal shock wave lithotripsy [E] is effective in breaking stones into small particles, but does not prevent stone recurrence.

6. Answer A

The patient is manifesting evidence of systemic inflammatory response syndrome [SIRS]; the presentation is most concerning for a pancreatic abscess. When SIRS is diagnosed, the first steps include fluid administration, blood cultures, and prompt institution of intravenous antibiotics [within 1 h], preferably imipenem. This should be followed by a CT scan [B] with contrast looking for necrotic tissue [non-enhancing areas] and a possible pancreatic abscess. In the past, such a finding on CT would warrant immediate exploration for pancreatic debridement [D]. However the current approach is termed a “step-up” approach, which consists of a series of increasingly more invasive interventions. This begins with a percutaneous attempt at drainage of any infected pancreatic collections. If that fails, the next step is laparoscopic drainage [E]. Finally open surgical pancreatic debridement is recommended. ERCP [C] would be indicated for suspected acute cholangitis, usually in association with gallstones.

7. Answer C

Persistent abdominal pain, fevers, and nausea beyond a few days following laparoscopic cholecystectomy should raise suspicion of a bile duct injury or a bile leak from the cystic duct stump [due to the surgical clip inadvertently coming off]. Imaging by CT scan should be obtained to look for a fluid collection. Abdominal ultrasound is also an acceptable imaging modality, but CT provides more information about the amount of fluid and its location. Additionally, findings on CT will guide subsequent management. Endoscopic ultrasound [A] is primarily used in the setting of pancreatic or bile duct cancer to help determine resectability and look for adjacent enlarged lymph nodes. If the patient has evidence of infection, and a large fluid collection is found, a percutaneous drain should be placed. Bilious output suggests that bile has leaked [from the stump of the ligated cystic duct or worse from an injury to the common hepatic/bile duct]. A HIDA scan [E] should be obtained next. If the common bile duct or common hepatic duct were inadvertently transected, the HIDA will show extravasation of tracer in the RUQ without tracer filling the small bowel. Such a finding would mandate exploratory laparotomy [B], and a loop of small bowel would need to be anastomosed to the proximal bile duct [hepaticojejunostomy]. If on the other hand, the HIDA scan shows extravasation of tracer in the RUQ but tracer is seen in the small bowel, this confirms that the integrity of the main bile ducts. The most common cause for this latter finding is a cystic duct stump leak [as in the present case]. Management is to perform ERCP with stenting [D] of the ampulla. This lowers the pressure in the biliary tree, creating a path of least resistance for the bile, thus permitting the cystic duct stump to seal.

8. Answer C

Patients with mild pancreatitis can often be managed being NPO along with intravenous hydration alone since recovery occurs rapidly, within 5–7 days, at which time oral intake can resume. However, patients with moderate-severe pancreatitis are unlikely to resume oral intake within 5–7 days, prompting the need for nutritional support. The most appropriate management is enteral nutrition. Enteral nutrition is provided through a nasojejunal tube, ideally placed past the ligament of Treitz as to not stimulate and irritate the pancreas. Enteral nutrition is preferred for those with a prolonged course of pancreatitis because it helps maintain the intestinal barrier and prevents bacterial translocation from the gut. In addition, enteral nutrition avoids the complications associated with parenteral nutrition including those secondary to venous access and blood stream infections. A 2010 meta-analysis of eight trials demonstrated that enteral nutrition significantly reduced mortality, multiorgan failure, infections, and the need for surgery as compared with those who received parenteral nutrition. Parental nutrition should only be initiated in patients who do not tolerate enteral feeding. In moderate-severe pancreatitis, oral feeding is not tolerated due to pain, nausea, or vomiting related to inflammation and edema causing gastric outlet obstruction, and should not be used.

9. Answer A

Gallbladder polyps are usually incidental findings, most are asymptomatic, and the vast majority are benign. Risk of malignancy is related to polyp size. Polyps 50 %]. Although a pulmonary embolism [B] may also present with hypoxia and a respiratory alkalosis, it is unlikely to present with bilateral infiltrates [the CXR is usually negative]. Hospital-acquired pneumonia [C] would present with a productive cough, dyspnea, chills, pleuritic chest pain, decreased breath sounds, wheezing, and a CXR showing lobar consolidation. A patient with pulmonary edema [D] would have a CXR showing cephalization of vessels and Kerley B lines [i.e., prominent horizontal interstitial markings in lower lung fields], as well as an elevated CVP [>18 mmHg]. Atelectasis [E] would appear as a white out of a lobe.

11. Answer B

Obstructive jaundice is an elevation of serum conjugated bilirubin due to inability to excrete it into the intestines via the biliary system. Thus there would be decreased available bilirubin in the intestine for intestinal bacteria to convert to urobilinogen and subsequently stercobilin. Although indirect and direct bilirubin are both elevated in obstructive jaundice, the direct component should be higher. Unconjugated bilirubin is bound to protein, and not filtered by the kidney. In patients with jaundice, conjugated bilirubin is elevated in the urine. The elevation in urine conjugated bilirubin gives it the brown [“tea-colored”] discoloration. Elevation of transaminases out of proportion to alkaline phosphatase [E] would suggest hepatocellular injury.

12. Answer B

Smoking is a risk factor for pancreatic cancer. Pancreatic cancer is also more common in men, advanced age, in the obese, and in African Americans. Chronic pancreatitis is the strongest risk factor pancreatic cancer, although alcohol consumption [A] per se is not. A family history of prostate cancer [C] is not a risk factor for pancreatic cancer. Although malabsorption [D] and pancreatic enzyme supplementation [E] is frequently associated with patients that have chronic pancreatitis, they are not themselves directly linked to an increased risk for pancreatic cancer.

13. Answer E

UGI bleeding from varices most often are the result of alcohol-related liver cirrhosis with subsequent portal hypertension. This leads primarily to esophageal varices and less commonly to concomitant gastric varices. Isolated gastric varices are uncommon. They often arise in association with splenic vein thrombosis [SVT], which forces all the venous drainage of the spleen to travel through the short gastric veins resulting in large gastric varices that are at risk for rupture and bleeding. The most common cause of SVT is pancreatitis [acute and chronic]. The most common cause of chronic pancreatitis is alcohol abuse. Peripancreatic inflammation can lead to occlusion of the splenic vein, which is posterior to the pancreas. SVT does not lead to esophageal varices because the collaterals do not involve the esophageal vasculature. Diagnosis of SVT can be made by duplex ultrasound of the splenic vein. It can also be detected on a venous phase CT scan. Splenectomy effectively eliminates the enlarged short gastric veins and thus cures the gastric varices. Gastric varices are particularly dangerous as they tend to cause massive bleeding. In addition, they do not respond well to standard treatment for esophageal varices such as banding [B] or sclerotherapy [C]. Both liver transplantation [A] and TIPS [D] would reduce portal hypertension and thus help remedy esophageal varices but would be ineffective for isolated gastric varices in the setting of SVT.

14. Answer C

The first step in working up a pancreatic leak is to test the drained fluid for amylase. In addition, serum amylase level should be checked as well. If fluid amylase level is high and output levels are high [>50 cm3/day], patient can be made NPO to decrease secretion of pancreatic fluid that accompanies oral intake. Imaging [A–B] can be done at a later time to evaluate adequacy of drainage. Octreotide [D] can be given to decrease pancreatic secretions, but it is not used routinely, and there is no evidence-based data that demonstrate cost-effective efficacy of octreotide use in this setting. TPN [E] is not routinely needed in this setting.

15. Answer D

At this time CA 19–9 is not recommended for screening pancreatic cancer, and is also not a diagnostic test. However, many surgeons use CA 19–9 to monitor for recurrence following surgery.

16. Answer B

The patient presented has RUQ pain and tenderness, nausea, and an ultrasound demonstrating gallstones with a thickened gallbladder wall suggestive of acute cholecystitis. Mild elevations in ALT and AST can be expected with acute cholecystitis, as well as leukocytosis with a left shift. The patient should be admitted to the hospital, made NPO, given IV fluids, and IV antibiotics with gram negative, enterococcus, and anaerobic coverage. She should undergo laparoscopic cholecystectomy, ideally within 48 h [C–E]. Multiple studies have shown that delaying surgery is of no benefit and, in fact, makes the operation technically more challenging due to more scarring. Elective outpatient laparoscopic cholecystectomy is appropriate for patients with biliary colic [A].

17. Answer A

This patient has acute pancreatitis, most likely secondary to gallstones [most common cause]. She has had prior episodes of pain after eating fatty food, which is characteristic of symptomatic gallstones. In addition, the lipase and amylase are elevated [3× greater than the upper limit of normal]. Cholecystectomy is considered the standard of care in patients with gallstone pancreatitis because there is a high risk of recurrent pancreatitis. Although most centers have traditionally waited until all laboratory values have normalized for patients with mild disease [may take 5–7 days] [B], recent randomized studies have demonstrated that cholecystectomy can be safely performed within 48 h of admission [in patients with mild pancreatitis] regardless of whether laboratory values have normalized [D]. Thus this patient should proceed with LC with IOC since she demonstrates no evidence of severe pancreatitis. On the other hand, with severe pancreatitis, such as necrotizing pancreatitis, delaying gallbladder removal until complete resolution of the pancreatitis is recommended. The gallstones that cause pancreatitis are usually small, and as such, in the majority of cases, the stone remains impacted very briefly, only transiently obstructing the ampulla of Vater, and soon after passes into the duodenum. As such persistent of a CBD stone is uncommon, and therefore ERCP is not usually needed [C, E].

18. Answer A

A diabetic patient presenting with symptoms suggestive of acute cholecystitis with high fevers, high WBC with left shift, and RUQ ultrasound demonstrating gas bubbles within the wall of the gallbladder should be suspected of having emphysematous cholecystitis, which is generally considered a surgical emergency. It is generally caused by gas-forming organisms, such as Clostridia and E. coli. Compared to acute cholecystitis, emphysematous cholecystitis is associated with a much higher mortality due to severe sepsis as the gallbladder becomes gangrenous. Broad-spectrum antibiotics [include high-dose penicillin or clindamycin to cover Clostridia] and fluid replacement are started immediately to stabilize the patient [B], but because of the risk of gangrene, these patients should undergo an immediate cholecystectomy. The diagnosis can be supported with CT scan of the abdomen [D] which will also demonstrate gas within the gallbladder wall. ERCP [E] is not indicated. Cholecystostomy [C] would not be an appropriate intervention for emphysematous cholecystitis as the necrotizing tissue infection would not be removed.

19. Answer C

A heavy calcified gallbladder is termed a porcelain gallbladder and is most commonly found incidentally on imaging for unrelated reasons. Patients are often asymptomatic. However, it is important to recognize that a porcelain gallbladder is associated with an increased risk of gallbladder adenocarcinoma. As such, the recommendation is that patients should undergo surgical management with laparoscopic cholecystectomy. Reassurance [A] or “watchful waiting” with annual CT scan [B] is not appropriate, even for asymptomatic patients, because of the risk for malignancy. There is no reason to suspect hyperparathyroidism, and so a serum calcium and PTH level would not be appropriate [D]. ERCP with brushings is useful for suspected bile duct cancer, but not for suspected gallbladder cancer [E].

Lower Gastrointestinal

Questions

  1. A 55-year-old male has been receiving serial ultrasound examinations to follow his abdominal aortic aneurysm [AAA]. Over the past year, the aneurysm has rapidly enlarged to 5.8 cm, and he undergoes endovascular abdominal aortic aneurysm repair [EVAR]. The operation itself is uneventful. However, on postoperative day 1, the patient develops a low-grade fever, left lower quadrant pain, and diarrhea that appears to be blood tinged. On physical examination, he has mild to moderate tenderness in the left lower quadrant without rebound or guarding. What is the next step in the workup?

    • [A]

      CT scan of the abdomen and pelvis with oral and IV contrast

    • [B]

      Exploratory laparotomy

    • [C]

      Formal mesenteric arteriography

    • [D]

      Flexible sigmoidoscopy

    • [E]

      Abdominal ultrasound

  2. A 30-year-old man with colon cancer secondary to familial adenomatous polyposis [FAP] arrives for follow-up after receiving a total proctocolectomy with end ileostomy. He was found to have colon cancer after presenting at the age of 27 with unexplained rectal bleeding, diarrhea, and abdominal pain. Subsequent colonoscopy found multiple adenomatous polyps in his colon. He has a 5-year-old son, who is screened and is positive for the APC gene. What is the recommended screening for his son?

    • [A]

      Colonoscopy starting at age 20

    • [B]

      Flexible sigmoidoscopy starting at age 10

    • [C]

      Colonoscopy starting at age 50

    • [D]

      Annual fecal occult blood test

    • [E]

      Annual barium enema

  3. A 66-year-old male presents with a large volume of maroon-colored stools combined with red blood. In the ED his blood pressure is 100/60 mmHg, and heart rate is 120/min. Physical examination is unremarkable. Two large bore IVs are inserted, and 2 liters of normal saline are given, after which the patient’s vital signs normalize. Laboratory tests are sent, including a type and cross. What is the next step in the management?

    • [A]

      Administer two units of O negative blood

    • [B]

      Place NG tube for aspiration

    • [C]

      Colonoscopy

    • [D]

      Exploratory laparotomy

    • [E]

      Place central line

  4. A 27-year-old man arrives to the emergency department complaining of bloody diarrhea and rectal urgency. He reports a normal appetite, and has not lost any significant weight. After initial workup yields no findings, he is referred to a gastroenterologist for a colonoscopy. He is found to have pseudopolyps in his colon, and subsequent biopsy results confirm ulcerative colitis. He is started on corticosteroids and sulfasalazine, which is able to control his symptoms. Which of the following is true regarding colon cancer and screening in patients with ulcerative colitis?

    • [A]

      Screening for colon cancer is not necessary

    • [B]

      Screening colonoscopy with random biopsies 8 years after disease onset

    • [C]

      Screening colonoscopy with biopsy only if a suspicious polyp is seen

    • [D]

      Screening colonoscopy annually once diagnosis is established

    • [E]

      The risk of colon cancer is much less than with Crohn’s disease

  5. A 50-year-old man comes to the doctor complaining of painful defecation. On occasion, blood is found on the toilet paper after wiping. He has been experiencing this problem for months but felt too embarrassed to seek medical attention. He has a history of constipation and has tried multiple stool softeners but to no avail. Rectal examination shows enlarged anal papillae with an edematous and hypertrophic skin tag in the posterior anal midline, most suggestive of a chronic fissure. He is referred to a surgeon and scheduled to undergo a lateral internal sphincterotomy. What is the most common complication of this procedure?

    • [A]

      Bleeding

    • [B]

      Anorectal sepsis

    • [C]

      Fecal incontinence

    • [D]

      Fistula formation

    • [E]

      Nonhealing of fissure

  6. An 88-year-old woman with multi-infarct dementia undergoes a coronary artery bypass with the left internal mammary artery used as a conduit for three-vessel disease. Her operation is a success, and she is transferred to the ICU to recover. She is receiving opioids for pain relief. Over the next few days, she develops a markedly distended abdomen with no bowel sounds, pain, or rigidity. She has also not had a bowel movement for the past 48 h. Rectal examination does not demonstrate any retained stool. KUB demonstrates markedly distended colon with gas in the rectum, without air fluid levels. What part of the large bowel is the most likely to perforate?

    • [A]

      Cecum

    • [B]

      Transverse colon

    • [C]

      Sigmoid colon

    • [D]

      Rectum

    • [E]

      There is very little risk of perforation

  7. A 75-year-old man arrives to the ED with abdominal pain. He has never experienced this before but reports having left-sided abdominal pain over the last 2 days that is relieved temporarily after defecation. He has a temperature of 101.2 °F with a blood pressure of 142/88 mmHg and pulse of 88/min. His laboratory examination is significant for a WBC of 14 × 103/μL [normal 4.1–10.9 × 103/μL] with 10 % bands. CT imaging reveals focal sigmoidal wall thickening with significant paracolic inflammation. He was treated as an in-patient for 3 days and discharged home without complications. Which of the following is the most appropriate follow-up option?

    • [A]

      High-fiber diet alone

    • [B]

      CT scan 2 weeks after resolution

    • [C]

      Elective sigmoid colectomy

    • [D]

      Barium enema

    • [E]

      Colonoscopy

  8. A 28-year-old woman presents to the ED because of abdominal pain that started 2 h ago. She has nausea and vomited twice in the last hour. She reports that the pain is predominantly in the right lower quadrant. On physical exam, her blood pressure is 120/60 mmHg, heart rate is 100/min, and she is afebrile. She is very tender in the RLQ on palpation at McBurney’s point and has pain on passive extension of the right hip. Her WBC is 14 × 103/μL [normal 4.1–10.9 × 103/μL], hemoglobin is 12 g/dL [12–15.2 g/dL], hematocrit is 36 % [37–46%], and platelet count is 250,000 [140–450,000]. Her urinalysis shows 1+ white blood cells. Which of the following is the best next step in management?

    • [A]

      Admit for observation

    • [B]

      CT scan of the abdomen

    • [C]

      Laparoscopic appendectomy

    • [D]

      B-hCG

    • [E]

      Broad-spectrum antibiotics

  9. A 38-year-old presents with symptoms and signs of acute appendicitis and undergoes laparoscopic appendectomy. At surgery the terminal ileum and cecum appear to be red and inflamed. The appendix is removed uneventfully. Final pathology of the appendix demonstrates no evidence of acute appendicitis. Two weeks later, he presents back to the ED with feces draining from his right lower quadrant wound. Which of the following is the most likely explanation for why the drainage may not spontaneously stop?

    • [A]

      A distal colonic obstruction

    • [B]

      Chronic inflammation

    • [C]

      An occult intra-abdominal abscess

    • [D]

      A retained sponge in the abdomen

    • [E]

      A missed malignancy

  10. Which of the following is most likely to require urgent colectomy?

    • [A]

      Cecal volvulus

    • [B]

      Sigmoid volvulus

    • [C]

      Acute diverticulitis

    • [D]

      Cecal adenocarcinoma

    • [E]

      Pseudomembranous colitis

  11. A 55-year-old man starts a new security job that requires a physical exam by a doctor. He has no past medical history, and everyone in his family is healthy. He has a well-balanced diet and exercises every day. He is evaluated during a routine examination which includes a discussion of health maintenance issues. He is surprised to hear that it is recommended for a man of his age to get a colonoscopy. He asks if there are any other options available. Which of the following is an appropriate recommendation with respect to this health maintenance screening strategy?

    • [A]

      Fecal occult blood test [FOBT] every 5 years

    • [B]

      Annual rectal examination

    • [C]

      Flexible sigmoidoscopy every 5 years + FOBT every 3 years

    • [D]

      Annual CT colonography

    • [E]

      Annual CEA levels

  12. A 90-year-old man with Alzheimer’s disease arrives from a nursing home with abdominal distention and pain for the past 12 h. He takes hydrochlorothiazide, donepezil, aspirin, and docusate. His physical exam reveals a temperature of 102 °F, blood pressure 90/70 mmHg, and pulse 112/min. His abdomen is rigid and diffusely tender with rebound and guarding. Laboratory examination is significant for WBC of 15 × 103/μL [normal 4.1–10.9 × 103/μL], BUN of 25 mg/dL [7–21 mg/dL], and creatinine of 1.8 mg/dL [0.5–1.4 mg/dL]. X-ray shows a markedly distended colon with a bent-inner tube sign. Following IV fluids and antibiotics, what is the best next step in management?

    • [A]

      Decompression with colonoscopy

    • [B]

      CT scan of the abdomen and pelvis

    • [C]

      Exploratory laparotomy

    • [D]

      Placement of a rectal tube

    • [E]

      Admit to hospital for close observation

  13. A 44-year-old man presents with right lower quadrant pain without rebound, nausea, and vomiting. CT scan shows bowel wall thickening near the ileocecal valve. He is scheduled for an appendectomy. Final pathology confirms acute appendicitis. In addition, an incidental 2.5 cm carcinoid tumor is found at the tip of the appendix. What is the most appropriate next step in management?

    • [A]

      Observation

    • [B]

      Colonoscopy within the next 6 months

    • [C]

      UGI with small bowel follow through

    • [D]

      Chest X-ray

    • [E]

      Right hemicolectomy

  14. A 64-year-old male returns for follow-up six months after successful sigmoid colectomy for colon cancer. Which of the following can lead to a falsely elevated serum CEA level?

    • [A]

      Chronic wound infection

    • [B]

      Smoking

    • [C]

      Hyperglycemia

    • [D]

      Age

    • [E]

      Postoperative chemotherapy

  15. A 4-year-old boy presents to the emergency department with right lower quadrant pain, nausea, and anorexia. On physical examination, his bowel sounds are absent, and he has marked tenderness in the right lower quadrant with guarding. The remainder of the abdominal exam is negative. Laboratory values reveal a leukocytosis with a left shift. What is the most likely initiating event for his acute condition?

    • [A]

      Lymphoid hyperplasia

    • [B]

      Fecalith

    • [C]

      Parasitic infection

    • [D]

      Enlarged mesenteric lymph nodes

    • [E]

      Foreign body ingestion

  16. A 50-year-old woman undergoes screening colonoscopy. During the procedure, she is found to have a dark discoloration of the entire colon with lymph follicles shining through as pale patches. What is the most likely underlying etiology?

    • [A]

      Drinking lots of prune juice

    • [B]

      Colon cancer

    • [C]

      Laxative abuse

    • [D]

      Normal anatomic variation

    • [E]

      Inflammatory bowel disease

  17. A 88-year-old man with Parkinson’s disease and COPD is brought to the ED from a skilled nursing facility because his nurse noticed bright red blood in his adult diapers. His medications include hydrochlorothiazide, metformin, levodopa, salmeterol, and docusate. On admission, he is afebrile with normal blood pressure and pulse. Nasogastric tube [NGT] aspiration returns yellow bile. Colonoscopy shows bright red blood within the colon, multiple diverticula, but due to the large amount of stool and blood clots, no active bleeding sources are able to be seen. He continues to have blood per rectum over the next hour. Blood pressure is 120/70 mmHg and heart rate is 90/min. Which of the following is the most appropriate next step in management?

    • [A]

      Exploratory laparotomy

    • [B]

      Transfuse two units of packed red blood cells

    • [C]

      99mTc red blood cell [RBC] scintigraphy

    • [D]

      Esophagogastroduodenoscopy [EGD]

    • [E]

      Omeprazole and antibiotics

  18. A 62-year-old female arrives to the ER with acute abdominal pain. She has a past medical history significant for diverticulitis. She is diagnosed with uncomplicated diverticulitis and managed with bowel rest and antibiotics. A year later, she has another episode of diverticulitis that was again successfully managed on an outpatient basis. Which of the following is this patient at higher risk for developing?

    • [A]

      Crohn’s disease

    • [B]

      Ulcerative colitis

    • [C]

      Colon cancer

    • [D]

      Anal cancer

    • [E]

      Stricture formation

  19. Endocarditis secondary to which of the following organisms is associated with colon cancer?

    • [A]

      Streptococcus bovis

    • [B]

      Clostridia septicum

    • [C]

      Steptococcus bovis and Clostridia septicum

    • [D]

      Diphyllobothrium latum

    • [E]

      Cryptococcus neoformans

Answers

1. Answer D

Ischemic colitis is one of the complications that can occur after AAA repair. The etiology is thought to be due to the fact that the inferior mesenteric artery [IMA] is ligated [with open repair] or occluded [with endovascular repair]. In the majority of patients, there is no consequence from ligating the IMA. Depending on collateral blood supply, the left colon may develop mucosal or full-thickness ischemia. Patients present with varying degrees of left lower quadrant pain, fever, diarrhea, or frankly bloody stool. Ischemic colitis is confirmed by flexible sigmoidoscopy that will demonstrate inflamed, friable mucosa, or full-thickness necrosis [bear in mind that a scope within the lumen cannot necessarily determine full-thickness necrosis]. Treatment of ischemic colitis begins with placing the patient NPO and administering IV fluids and broad-spectrum antibiotics. If there is evidence of sepsis and/or peritonitis, the patient will require exploratory laparotomy [B], colonic resection, and a proximal colostomy. Since the cause of the ischemic colitis [ligation of the IMA] is known, and cannot be reversed, CT [A], formal mesenteric arteriography [C], and ultrasound [E] are not helpful.

2. Answer B

Familial adenomatous polyposis [FAP] is an autosomal dominant condition in which patients develop hundreds to thousands of polyps in the colon, which if left untreated, has a 100 % chance of developing into cancer by age 50. The mutation is in the adenomatous polyposis coli [APC] gene on chromosome 5. A child who has a parent with the mutation has a 50 % chance of inheriting the syndrome. Thus APC gene testing is recommended. If the child tests positive, screening with flexible sigmoidoscopy should begin at age 10 [as polyps can develop even in the teen years]. Once a polyp is seen, surgery is recommended to remove the colon. Since the polyps begin on the left side of the colon, a full colonoscopy [A, C] is not required for screening. Annual fecal occult blood test [D] or barium enema [E] are not adequate screening tools in FAP.

3. Answer B

Prior to initiating a workup for GI bleeding, it is important to determine if the source is the upper or lower GI tract. If the patient is vomiting blood or has coffee ground emesis, then the source is clearly an UGI one. Since a massive UGI bleed can cause bleeding per rectum, it is imperative that an UGI source is ruled out first. This is achieved by placing an NG tube and aspirating for blood or coffee grounds. If it returns blood, an upper endoscopy should be performed first. If the aspirate returns bile, then an UGI bleed is ruled out. Since the patient responded to a fluid challenge, a blood transfusion [A] is not necessary. Colonoscopy [C] is not yet indicated nor is surgery [D]. A central line [E] should be considered if ongoing massive fluids/blood are needed.

4. Answer B

Patients with IBD are at increased risk of colon cancer. The risk is much greater for ulcerative colitis [UC] than with Crohn’s and is related to the duration of illness and the extent of disease. For UC, the risk is low in the first 10 years of the disease [2–3%] but grows to 1–2% per year afterwards. UC patients should begin getting screened 8 years after disease onset and continue getting screened every 1–2 years after. Random biopsies are necessary in patients with UC undergoing screening with colonoscopy because in these patients, cancers do not follow the typical progression from polyp to cancer. A proctocolectomy removes the entire rectum and colon, which prevents patients with UC from developing cancer and no further surveillance is required.

5. Answer C

Anal fissure is the most common cause of minor painful rectal bleeding [hemorrhoids usually cause minor painless bleeding]. To get an anal fissure, one needs two things: anal trauma [from hard stools] and a hypertonic/hypertrophied internal sphincter. The hard stool tears the anoderm, most often in the posterior midline [most vulnerable to damage due to a relatively diminished blood supply]. Patients typically present with painful defecation and blood found on tissue after wiping. Chronic fissures become deeper and will have enlarged anal papillae with hypertrophic and edematous skin tags [sentinel pile]. Medical management includes sitz baths [relaxes sphincter], fiber, and stool softeners. If this treatment fails, nitroglycerine ointment [limited by side effect of headaches] and botulinum toxin [injected into the internal sphincter] are the next steps in management. Patients that fail medical therapy can be considered for surgical treatment with lateral internal sphincterotomy, which is able to cure 95 % of cases. The risk of incontinence ranges from 5 % to 15 %. The other complications listed are uncommon [A–B, D–E].

6. Answer A

Ogilvie’s syndrome is a pseudo-obstruction of the colon that is associated with bedridden, neurologically impaired or older patients, opiate use, recent surgery, trauma, or infection. The mechanism for this colon dysfunction is unknown. X-ray films will predominantly show a markedly colon without evidence of a bowel obstruction. Management consists of removing any drugs that may interfere with gut motility [e.g., opiates] and replacement of electrolytes [especially potassium]. A colon larger than 10 cm is at risk for perforation and requires decompression with colonoscopy and neostigmine. Due to the law of Laplace [tension = pressure x radius], the cecum, being the part of the colon with the largest diameter, is the most common site for perforation [B–E].

7. Answer E

Diverticula occur as a result of herniation of mucosa through the colonic wall at sites where arteries enter the muscular layer. The incidence of diverticula increases with age, and it is believed to be present in 35 % of the population. When diverticula get infected [diverticulitis], often on the left side, patients can present with fever, leukocytosis, and LLQ pain that may be temporarily relieved following defecation. This patient has a moderate case of diverticulitis given the CT findings of significant inflammation in the sigmoid colon, fever, and leukocytosis. Treatment initially consists of bowel rest and IV antibiotics. Patients with acute diverticulitis should receive a follow-up colonoscopy to rule out malignancy [as the CT cannot readily distinguish diverticulitis from colon cancer]. Colonoscopy should not be performed during the same hospitalization as the insufflation of air may lead to free perforation. Barium enema [with or without sigmoidoscopy] [D] is not a recommended screening tool for colon cancer. In addition to a colonoscopy, a fiber-rich diet [A] should be encouraged for all patients with diverticulitis to reduce the incidence of diverticula. Follow-up CT scan [B] is unlikely to provide any additional information other than confirming diverticula. Surgery [C] is recommended for cases with significant complications [e.g., complete obstruction, free perforation with diffuse peritonitis] and those that have failed medical management.

8. Answer D

The differential diagnosis of RLQ pain in women is more extensive than for men and should include ruptured ectopic pregnancy, ruptured cyst, ovarian torsion, and pelvic inflammatory disease. A B-hCG test should always be ordered in a woman of child-bearing age with abdominal pain to rule out pregnancy. If positive, a ruptured ectopic pregnancy should be high on the differential. Although the patient presented has McBurney’s point tenderness and an elevated WBC, laparoscopic appendectomy [C] would not be recommended as of yet until the B-hCG is obtained. In addition, in women, ultrasonography is highly useful to rule out the aforementioned differential. CT scan [B] is not necessary to confirm the diagnosis of acute appendicitis. Rather it is used if the diagnosis is in question. Admission for observation [A] would not be appropriate. Do not assume that pyuria rules out appendicitis [an inflamed appendix can cause sterile pyuria].

9. Answer B

A rare complication after appendectomy is a cecal fistula. The findings on laparoscopy [inflamed terminal ileum and cecum] combined with a normal appendix indicate that the patient’s actual diagnosis is likely Crohn’s disease which can mimic appendicitis [in this setting, it is termed regional enteritis]. If regional enteritis is found instead of appendicitis at the time of laparoscopy, the appendix is removed even if it looks normal, provided the cecum is not involved in the inflammation. This prevents confusion in case the patient presents again with RLQ pain in the future. However, if the cecum is also inflamed in the setting of regional enteritis, the appendix should not be removed, as there is a risk that the stump of the appendix will blow out and form a fistula [as in the present case]. Conditions that prevent fistulas from spontaneously closing can be remembered by the acronym “HIS FRIEND”: high fistula output [>500 cm3/day], inflammatory bowel disease, short fistula [50,000] would be suggestive of septic arthritis [A, B]. Rheumatoid factor [C] found in synovial fluid would be supportive of a diagnosis of rheumatoid arthritis, which is treated systemically. Finally, negative birefringent crystals [D] are the hallmark of gouty arthritis, which is not treated surgically.

2. Answer A

The patient is most likely exhibiting slipped capital femoral epiphysis [SCFE], which affects obese adolescent males. The physical exam finding being described is the Trendelenburg sign, which is a shift of the torso over the affected hip due to gluteus muscle weakness. In addition, physical exam may reveal that the patient’s gait is altered with the patient taking a short step on the affected side to minimize weight bearing due to pain. The anterior hip may be tender to palpation. Despite a complaint of pain in the knee area, the knee examination is normal [D, E].

3. Answer C

Anterior dislocations of the humeral head increase the patient’s risk for concurrent axillary nerve injury. All these patients should be evaluated for axillary nerve damage. This can cause paralysis of the deltoid and teres minor muscles as well as a loss of sensation over the upper lateral arm. Musculocutaneous nerve injuries [A] are not common in patients that have sports injuries. These patients will present with paralysis or weakness in the biceps and brachialis muscles. Radial nerve injuries [B] are associated with fractures of the humeral midshaft. Brachial arterial injury [D] is a concern for patients with a supracondylar fracture. Medial clavicle injury [E] is difficult to attain and is not expected to occur concurrently in patients that have an anterior dislocation of the humeral head.

4. Answer E

Severed body parts can sometimes be reattached if the proper steps to ensure tissue viability are taken. There are new and promising replantation techniques available. However, nerve regeneration continues to be a major limiting factor. Young patients with sharp amputations and no crush injury are the best candidates for replantation. The best recommendation to keep an amputated digit viable is to remove any dirt from the digit first, wrap it in a clean damp cloth, put it in a plastic bag, and then place the bag directly in a cup of cold water. This will ensure viability for up to 18 h.

5. Answer A

The most consistent finding with a tear in the posterior cruciate ligament would be a lower leg that sags on passive flexion of the knee at 90° while the patient is supine. An ACL tear would present with the lower leg moving forward relative to the knee with forward traction [B]. A tear in the MCL would present with a foot that moves laterally when the knee is pushed medially [C]. A tear in the LCL would present with a foot moving medially when the knee is pushed laterally [D]. A clicking sensation that is heard when the knee is flexed and the leg is rotated externally is known as McMurray’s sign and is positive with a meniscal tear [E].

6. Answer B

Radial nerve injuries present with wrist drop and sensory loss to the posterior arm and lateral dorsal aspect of the hand. This most commonly occurs in patients that have had fractures to the humeral midshaft and those that use improperly fitted crutches. This patient is using crutches that belonged to his older brother and is likely responsible for his upper extremity abnormalities. If a fracture in the midshaft of the humerus [A] did not present clinically during the time of his snowboarding accident, it is unlikely to cause significant problems a month later. Long thoracic nerve injuries [C] can occur in women with breast cancer that receive axillary lymphadenectomy. This will present with scapular winging. Ulnar nerve injury [D] presents with sensory changes in the 4th and 5th digits. Musculocutaneous nerve injuries [E] will present with paralysis or weakness in the biceps and brachialis muscles.

7. Answer A

This patient has suppurative tenosynovitis with the characteristic four cardinal signs [Kanavel signs]: flexor tendon sheath tenderness, fusiform swelling [sausage-shaped digits], pain with passive extension, and a semi-flexed posture of the involved digit. It is a closed space infection, and some may be associated with past penetrating injuries to the hand. The most likely organism is Eikenella corrodens which is often associated with human bites [e.g., punch in the mouth/face]. Pasteurella multocida [D] can also cause suppurative tenosynovitis but is associated with cat scratches. Management of suppurative tenosynovitis involves mid-axial longitudinal incision and drainage. Gout [B] is a crystal-induced arthropathy that commonly first presents in the big toe [podagra]. Dupuytren’s contracture [C] is associated with diabetes, smoking, and alcohol abuse. It presents with contractures in the fourth and fifth digits secondary to proliferation of the palmar fascia of the hand. Felon [E] is a term used to describe infection in the terminal joint space of the finger.

8. Answer E

This patient is most likely exhibiting avascular necrosis of the proximal femoral head [hip] [Legg-Calvé-Perthes disease]. Avascular necrosis of the hip typically presents as hip pain, anterior thigh pain, or referred knee pain as well as a limp [which may be painless] in children between the ages of 4 and 10 years with a male-to-female ratio of 4:1. Often, children are unable to localize hip pain and may state they have knee pain, confusing the picture. Avascular necrosis is usually idiopathic but may occur secondary to an underlying condition such as glucocorticoid use. Clinical features include insidious onset of hip pain and limp. Physical exam shows limitation in internal rotation of the hip. Initial radiographs are often normal. Later in the course, radiographs show fragmentation of the femoral head. Septic arthritis presents with acute onset of refusal to bear weight, pain, swelling, warmth, with fever, and leukocytosis [A]. Osteomyelitis is spread hematogenously in children and affects the metaphysis of long bones. It presents with localized pain and fever along with leukocytosis and elevated ESR and CRP [B]. Slipped capitol femoral epiphysis [SCFE] affects obese adolescent males and presents with altered gait [C]. Torn ACL affects adolescent girls more often and would present with a history of trauma, swelling at the knee joint, and increased anterior translation of the knee on physical exam [D].

9. Answer A

This maneuver is called the drop arm test and is used to evaluate for a tear in the supraspinatus muscle, a rotator cuff injury. This is performed by passively abducting the patient’s shoulders to 90° and flexing to 30° while asking the patient to point his thumbs down. The test is positive if the patient is unable to keep arms elevated after the examiner releases. The supraspinatus is part of the rotator cuff, along with the infraspinatus [B], teres minor [C], and the subscapularis muscles. Teres major [D] is not part of the rotator cuff.

10. Answer B

Carpal tunnel syndrome is a clinical diagnosis and can be supported with a positive Tinel’s and/or Phalen’s sign. Tinel’s sign is elicited by gently percussing over the median nerve at the carpal tunnel. A positive sign is present if the patient described an electrical shock sensation in the median nerve distribution [A]. Phalen’s test is performed by having the patient place their elbow on a table and flexing the wrist for 60 s. The test is considered positive if the patient reports paresthesias in the median nerve distribution. The median nerve controls sensation to the thumb, index, middle, and half of the ring finger. Palmar sensation is not affected by carpal tunnel syndrome [C] because the superficial palmar cutaneous branch of the median nerve passes superficially to the carpal tunnel. Radial deviation of the wrist upon wrist flexion [D] would be expected with ulnar nerve injuries. Posterior and lateral forearm sensory deficit [E] would be expected with musculocutaneous nerve injuries.

11. Answer E

This infant is exhibiting developmental dysplasia of the hip [DDH], which describes a spectrum of conditions that cause the abnormal development of the acetabulum and proximal femur in infants and children. In young infants, it is important to routinely evaluate for DDH using the Barlow and Ortolani maneuvers, which are physical examination techniques to detect hip instability that use adduction and posterior pressure to feel for dislocatability and abduction to feel for reducibility, respectively. If a sensation of a “clunk” is found on physical exam, further work-up is warranted including immediate referral to an orthopedic surgeon. Reassurance and reexamination would be inappropriate [A, D]. Ultrasound is the primary imaging technique for assessing abnormalities of the hip until 4–6 months of age because plain X-rays have limited value in the first 6 months of life when the femoral head and acetabulum are not yet ossified [C]. CT scan is not useful is the diagnosis of DDH [B]. Treatment involves splinting, casting, and/or surgery.

12. Answer B

This patient has a classic history for a scaphoid fracture. This injury usually occurs in patients that fall on an outstretched hand with the wrist extended and presents with tenderness in the anatomic snuffbox. Plain films are typically unrevealing of scaphoid fractures if taken soon after injury. If clinical suspicion is high, all patients should be immobilized with a thumb spica cast and reimaged 7–10 days later. Supportive care with or without radiographs is not appropriate for classic cases of scaphoid fractures [A, D]. Radiographs are recommended for all patients [C] to look for more serious injuries that may be associated with scaphoid fractures. If patients are found to have signs suggestive of wide displacement or ligament disruption, then more advanced imaging studies should be done [E].

13. Answer A

Femoral neck fractures are most commonly seen in elderly patients after a fall. Women sustain hip fractures more often due to their higher rates of osteoporosis. Femoral neck fractures have a relatively high rate of complications compared with extracapsular hip fractures because they are intracapsular. The risk for compromise in the blood supply to the femoral neck as a result of the fracture increases the risk of complications, in particular avascular necrosis [AVN]. Long-term loss of hip abduction/adduction is prevented with physical rehabilitation [C–D]. Osteosarcoma [E] or lumbar radiculopathy [B] are not complications of a femoral neck fracture.

14. Answer D

An anterior dislocation of the shoulder joint is the most common form of shoulder dislocation. Posterior dislocations are rare and occur most often in patients that have had generalized seizures or have been electrocuted. Patients with posterior shoulder dislocations will present with an adducted arm that is internally rotated, while anterior dislocations present with an externally rotated arm. Sport injuries [A] are more likely to cause anterior dislocations of the shoulder joint. Similarly, a sudden forceful subluxation of the shoulder that may occur from a dog tugging at the chain [C] can lead to an anterior shoulder dislocation. Weight lifters are at increased risk for thoracic outlet obstruction which presents with symptoms caused by obstruction of nerves and vessels passing from the thoracocervical region to the axilla [e.g., upper extremity paresthesia, weakness, and edema]. Swimmers and surfers [E] are more likely to present with symptoms consistent with nerve impingement secondary to repetitive paddling.

15. Answer D

The patient is most likely exhibiting slipped capital femoral epiphysis [SCFE], which affects obese adolescent males. In SCFE, the femoral head separates from the neck and slips posteriorly, resulting in a limp and impaired internal rotation. Patients often present with knee pain, so a high index of suspicion is necessary to diagnose SCFE. The physical exam finding includes altered gait such that the patient takes a short step on the affected side to minimize weight bearing due to pain and tenderness to palpation at the anterior hip. The diagnosis of SCFE is made with X-rays, which show the classic “ice cream slipping off the cone” suggesting posterior displacement of the femoral head. Obesity seems to be the strongest risk factor for SCFE, likely due to excessive mechanical stress on the physis [growth plate]. Weight loss [C] can be beneficial to the overall health of the child and will decrease the risk of SCFE in the contralateral hip, but is not considered a definitive management for SCFE. Treatment involves operative stabilization using pinning of the hip joint. Supportive therapy with rest and ibuprofen would be the appropriate treatment for transient synovitis [A]. Aspiration of the synovial fluid along with appropriate antibiotic therapy would be the treatment for septic arthritis [B]. Pavlik harness [E] is used in the management of developmental dysplasia of the hip [DDH] to keep the hip in a flexed and abducted position.

16. Answer E

This patient has a meniscal tear with a positive McMurray’s sign. This maneuver helps identify a meniscal tear and is positive if there is a palpable or audible snap occurring when extending the knee from a fully flexed position while applying tibial torsion. Meniscal tears can also present concurrently with other injuries. A medially directed blow to the lateral knee [valgus stress] results in the “unhappy triad”: medial meniscus tear, medial collateral ligament tear, and anterior cruciate ligament tear. Both ligamentous [A, B] and meniscal tears can produce popping sounds during the injury, but ligamentous tears have rapid swelling occurring immediately, while meniscal tears have delayed swelling occurring the next day. Stress fractures [C] do not typically occur in the patella following trauma. Instead, they occur more commonly in long bones [e.g., tibia] that are subjected to repeat stress. A patellar tendon displacement [D] will present with a patient unable to resume weight bearing, an indentation at the bottom of the kneecap, a proximally displaced patella, and hemarthrosis.

17. Answer C

The most common cause of unilateral hip pain in the adult is inflammation of the trochanteric bursa. It is caused by friction of the gluteus medius tendon and the tensor fascia lata over the outer femur as a result of gait impairment, trauma, or infection. Untreated, the bursal wall thickens, fibroses, and gradually loses its ability to lubricate the outer hip. Hip osteoarthritis [A] most commonly presents in patients over 40 years of age. The principal symptom associated with osteoarthritis is hip pain, which is localized in the groin and exacerbated by activity and relieved by rest. Meralgia paresthetica is compression of the lateral femoral cutaneous nerve. The nerve is susceptible to compression. The principle symptom is numbness and tingling and/or burning pain localized in the upper outer thigh [B]. Osteonecrosis presents with groin pain [D]. Hip fracture would present with severe pain, inability to bear weight, and intolerance to movement of the hip [E].

18. Answer B

Osteosarcomas are primary malignant tumors of bone. The primary presenting symptoms of osteosarcoma is localized pain that is typically present for an extended period of time and may be worse at night. On physical exam, there may be a soft tissue mass, which is frequently large and tender to palpation. Osteosarcomas have a predilection for the metaphysis of long bones and are most commonly found in the distal femur, proximal tibia, and proximal humerus. The first diagnostic test to work-up bone pain is an X-ray, which shows periosteal bone formation, lifting of the cortex, and the classic “sunburst pattern.” Growing pains present with recurrent, self-limited extremity pain. In contrast to bone pain, which is unilateral, growing pains are bilateral. Ewing’s sarcoma presents in a similar manner, but the radiographic appearance is described as “onion peel” appearance due to the periosteum being displaced by the underlying tumor causing the characteristic periosteal reaction that produces layers of reactive bone [A]. Osteomyelitis presents with localized pain, fever, and leukocytosis of acute onset [C]. Osteochondroma [exostosis] [D] is a benign, firm solitary bone tumor commonly occurring in teenage males. Osteoid osteoma [E] is a benign, painful growth of the diaphysis in long bones [e.g., femur, tibia]. It also occurs in teenage males. The pain is characteristically worse at night but better with aspirin. On plain film, there is a central radiolucency surrounded by a sclerotic rim.

19. Answer A

A medially directed blow to the lateral knee [valgus stress] results in the “unhappy triad”: medial meniscus tear, medial collateral ligament tear, and anterior cruciate ligament tear. The most appropriate test to order in soft tissue injuries of the knee is a MRI. This study is the most appropriate study to investigate soft tissue injuries of the knee. It is superior to CT [B] as it is better able to demonstrate ligamentous and meniscal lesions. However, its use should only be limited for patients in which the diagnosis is in question. MRI is no more accurate than the physical examination of an experienced clinician, and so referral to a sport medicine physician or orthopedic surgeon should be considered prior to ordering a MRI. Radiographic studies [C] are not typically helpful or performed for sports injuries, but they are useful in traumatic knee injuries. Arthroscopy can be used for diagnostic purposes if the MRI is equivocal or if it is abnormal and warrants subsequent surgical intervention. Although nerve injuries may accompany knee injuries, nerve conduction studies [E] are not typically ordered in work-up of soft tissue injuries of the knee.

20. Answer C

This surgeon most likely has a tibial stress fracture. This injury is common in patients that have a sudden change in physical activity and/or long periods of standing. His fatigue could be explained by jet lag and returning from an intense medical mission trip. Stress fractures present with pain upon palpation and some surrounding edema of the skin. After physical exam, the first step in evaluating a possible stress fracture is a plain film. It is unlikely to show up on plain films, and so clinical judgment should dictate weather to start appropriate therapy. MRI or bone scintigraphy can be considered next to further evaluate the injury. Most patients with stress fractures could be managed with supportive therapy: reduce activity to the level of pain-free functioning, over-the-counter analgesics, stretching exercises, and/or biomechanical stress-relieving measures. MTSS [A], also known as shin splints, is a common cause of exertional leg pain in athletes. Patients with MTSS complain of vague, diffuse pain of the lower extremity, along the middle-distal tibia associated with exertion. Conversely, patients with exertional compartment syndrome [C] will present with exercise induced pain in the lower legs that quickly disappears with rest. This occurs in college athletes and long distance runners. The exact cause is not well understood, but several leading theories exist. During exercise, blood flow to the muscle increases, and in patients with constricted compartments [e.g., hypertrophic leg muscles in college athletes], this increased blood flow may result in increased pressures that can cause pain. Osgood-Schlatter disease [B] occurs in adolescence during a time when there is increased strain on the tibial tubercle [from repetitive quadriceps contraction via the patellar tendon]. Exertional pain at the knees that resolves with rest is the most common complaint. Osteomyelitis [E] is a possibility and can be ruled out with a serum ESR. However, a stress fracture is far more likely.

21. Answer A

Most clavicle fractures can be managed conservatively with a shoulder string or brace for 6–8 weeks. However, all patients with fractures to the clavicle should receive a careful neurovascular examination since these patients are at risk for brachial plexus and axillary/subclavian arterial injury. CT angiogram is not routinely necessary, but is indicated in the presence of a thrill or bruit around the clavicle, diminished or absent radial/brachial pulse, fracture of the first rib, large hematoma in the supraclavicular region, or mediastinum widening on plain films. A CT scan [B] of the shoulder can help estimate the intra-articular extension of the clavicular fracture in the acromioclavicular joint; however, contrast should be given to look for arterial injury. MRI [C] can help diagnose coexistent injuries of rotator cuff or intra-articular disk of the acromioclavicular joint. Although radial nerve injuries can commonly accompany midshaft humeral fractures, they are more likely to present clinically with wrist drop and sensory loss to the posterior arm and lateral dorsal aspect of the hand. Nerve conduction studies [D] are typically not needed, particularly when there are no physical exam findings suggestive of nerve damage. Blunt cardiac injury can occur in patients following MVC. This should be considered if the patient presents with chest pain and/or hemodynamic instability. Given his age, the bruit heard in his chest is unlikely to be related to underlying cardiovascular disease. For these reasons, an echocardiogram is not indicated [E].

22. Answer A

Pregnant patients with preeclampsia or eclampsia that present with an inability to move the arm following a seizure are most likely suffering from a posterior shoulder dislocation. These are rare and occur most often in patients that have had generalized seizures or have been electrocuted. Pregnant patients suffering from eclampsia are also at risk because of its association with seizures. Regular anteroposterior radiographs will often miss the diagnosis, and so patients suspected of having a posterior dislocation should receive axillary and lateral view radiographs. The axillary view is essential for diagnosis as it can help estimate the size of the defect in the humeral head. If radiographs are equivocal, a CT scan [C] can be ordered next. MRI [B] is considered for older patients with shoulder dislocation as they are more likely to have concurrent rotator cuff injury. Reassurance is not appropriate for shoulder dislocation [E]. Most patients are able to manage a shoulder dislocation with nonoperative therapy [D].

Pediatric Surgery

Questions

  1. A 4-week-old boy presents with a 3-day history of forceful vomiting. The mother states that the vomitus only contains partially digested milk. She notes that the infant seems very hungry between feedings and drinks vigorously. Past history is significant for a skin infection for which the infant received oral erythromycin. On examination, the infant appears to be healthy appearing and in no acute distress. The physician feels there may be a small palpable mass in the right upper quadrant, but is not certain. Electrolytes are normal. What is the best way to establish the most likely diagnosis?

    • [A]

      Plain abdominal x-rays

    • [B]

      Laparoscopy

    • [C]

      CT of the abdomen

    • [D]

      Upper GI study with contrast

    • [E]

      Ultrasound

  2. A 2-day-old male infant was diagnosed prenatally with Down syndrome. Delivery was uneventful, but pregnancy was complicated by polyhydramnios. The infant has had several episodes of bilious vomiting after breast feeding but is otherwise stable. Which of the following would be the most likely finding on further work-up?

    • [A]

      Dilated loops of small bowel with air-fluid levels on abdominal x-ray

    • [B]

      Inability to pass a nasogastric tube

    • [C]

      Abdominal distention with erythema of the overlying skin

    • [D]

      Two large air bubbles on abdominal x-ray

    • [E]

      Narrowing of the distal rectum on GI contrast study

  3. A 12-day-old male born at 33 weeks becomes lethargic and hypothermic over the course of 24 h. He is not tolerating his formula feeds, has two episodes of bilious emesis, and has three episodes of bloody diarrhea. Physical exam reveals abdominal distention, visible loops of bowel, abdominal wall erythema, and absent bowel sounds. What is the most likely diagnosis?

    • [A]

      Hirschsprung’s disease

    • [B]

      Duodenal atresia

    • [C]

      Esophageal atresia with tracheoesophageal fistula

    • [D]

      Necrotizing enterocolitis

    • [E]

      Meconium ileus

  4. A newborn female infant is born to a 19-year-old G1P0 mother who smokes. On physical exam, the small bowel is eviscerated through an abdominal wall defect to the right of the umbilicus. The small bowel appears matted and dilated. The infant appears to otherwise be healthy. Which of the following would have been expected in prenatal screening?

    • [A]

      Elevated b-hCG levels

    • [B]

      Decreased estradiol levels

    • [C]

      Elevated alpha-fetoprotein levels

    • [D]

      Oligohydramnios

    • [E]

      None of the above

  5. A 4-month-old baby girl is seen at her pediatrician’s office for her well-child check. The parents raise a concern that she has been vomiting approximately 1/3 of her meals since 2 weeks of age. The emesis is the color of milk and is not bile-stained. There has been no change in the frequency or amount of emesis. She is exclusively breast-fed. On physical exam, mucous membranes are moist and the anterior fontanelle is open and flat. Her growth is at the 75th percentile for height and weight and has not changed significantly since birth. She is otherwise asymptomatic and without findings on physical examination. Which of the following is the most likely diagnosis?

    • [A]

      Tracheoesophageal fistula

    • [B]

      Duodenal atresia

    • [C]

      Pyloric stenosis

    • [D]

      Gastroesophageal reflux

    • [E]

      Malrotation

  6. A 6-h-old male infant is noted to be dyspneic with an oxygen saturation of 86 %. Physical exam reveals subcostal retractions and moderate perioral cyanosis. Which finding on chest x-ray would be most suggestive of a nonsurgical diagnosis?

    • [A]

      Loops of bowel in the left chest

    • [B]

      Tip of orogastric tube located above carina

    • [C]

      Diffuse pulmonary interstitial edema

    • [D]

      Boot-shaped heart with upturned apex

    • [E]

      Double-bubble sign just beneath the diaphragm

  7. A newborn full-term infant is noted to have several episodes of bilious emesis. He is otherwise stable. Which of the following is the LEAST likely diagnosis?

    • [A]

      Pyloric stenosis

    • [B]

      Duodenal atresia

    • [C]

      Midgut volvulus

    • [D]

      Hirschsprung’s disease

    • [E]

      Jejunal atresia

  8. A 5-week-old boy presents with a 6-day history of vomiting. The parents report that his vomiting is more forceful than his usual spit ups and contain significantly more volume. The mother describes the vomit as partially digested milk. Between episodes of vomiting, the baby feeds vigorously. The parents report that he has made only one wet diaper today. On physical exam, the baby appears lethargic. Mucous membranes are dry and the anterior fontanelle is open and sunken. Capillary refill is 2 s. The growth chart reveals a 0.5-lb weight loss since his clinic visit 7 days ago. What is the next step in management?

    • [A]

      Surgical intervention

    • [B]

      CT of the abdomen

    • [C]

      Upper GI contrast study

    • [D]

      Fluid resuscitation

    • [E]

      Ultrasound

  9. A 2-h-old male infant born at 39 weeks gestation is noted to be drooling. Prenatal ultrasound demonstrated polyhydramnios. Attempts at placement of an orogastric tube are unsuccessful as the tube only passes about 10 cm from the lips. What is the most important immediate concern for an infant with this condition?

    • [A]

      Prevention of aspiration

    • [B]

      Nutrition

    • [C]

      Urgent surgical exploration

    • [D]

      Establishing positive pressure ventilation

    • [E]

      Confirmatory contrast esophagram

  10. Which of the following is the next best step in the management of a 1-week-old infant born at full term with bilious emesis?

    • [A]

      Abdominal ultrasound

    • [B]

      Broad-spectrum antibiotics and blood cultures

    • [C]

      Immediate operative repair

    • [D]

      Contrast enema

    • [E]

      IV fluids and nasogastric tube placement

  11. A newborn male is found to have the majority of his small bowel eviscerated through an abdominal wall defect. The umbilicus appears to be intact. There is no membrane covering the bowel. The most important immediate risk to an infant with this condition is related to:

    • [A]

      Sepsis

    • [B]

      Respiratory compromise

    • [C]

      Cardiac anomalies

    • [D]

      Dehydration

    • [E]

      Urinary obstruction

  12. A 6-week-old boy presents with a 6-day history of non-bilious, forceful vomiting. Between episodes of vomiting, the baby feeds vigorously. On physical exam, the baby has dry mucous membranes with a sunken anterior fontanelle. Capillary refill is 2 s. He is otherwise asymptomatic without findings on physical exam. Ultrasound is obtained and is shown below. What electrolyte abnormality would you expect?

    • [A]

      Hypochloric metabolic alkalosis

    • [B]

      Hyperkalemic metabolic acidosis

    • [C]

      Hyponatremic metabolism acidosis

    • [D]

      Hyperkalemic metabolic alkalosis

    • [E]

      Hyponatremic metabolic alkalosis

  13. A 2-h-old male born at 39 weeks gestation is noted to have diffi culty breathing. Which of the following would most strongly suggest a diagnosis of esophageal atresia with a tracheoesophageal fi stula?

    • [A]

      Worsening respiratory status with feeding

    • [B]

      Olive-shaped mass palpable in the epigastric region

    • [C]

      Improvement of respiratory status over the 24 h after birth

    • [D]

      Chest x-ray showing dilated loops of small bowel in the left hemithorax

    • [E]

      Scaphoid abdomen

  14. A 2-h-old male born at 39 weeks gestation with difficulty breathing is diagnosed with esophageal atresia with a tracheoesophageal fistula and undergoes surgical repair, which was uncomplicated. Ten years later, the patient develops difficulty swallowing and often vomits undigested food shortly after eating. Which of the following is the most likely explanation for this?

    • [A]

      Scleroderma

    • [B]

      Gastroesophageal reflux

    • [C]

      Esophageal cancer

    • [D]

      Esophageal leak

    • [E]

      Esophageal stricture

  15. A 2-week-old male infant born at 26 weeks gestation is in the neonatal ICU when he becomes hypotensive and begins passing bloody stools. Which of the following is the most likely finding?

    • [A]

      Palpable olive-shaped mass in the epigastric region

    • [B]

      Double-bubble sign on abdominal x-ray

    • [C]

      Loops of intestine in the left hemithorax on chest x-ray

    • [D]

      Gas in the walls of the intestine

    • [E]

      Donut sign on abdominal ultrasound

  16. Which of the following ventilator settings or respiratory measurements would be LEAST acceptable for a newborn with congenital diaphragmatic hernia [CDH]?

    • [A]

      100% FiO2

    • [B]

      PEEP of 20 cm H2O

    • [C]

      Inhaled nitric oxide [NO]

    • [D]

      PaCO2 of 55 mmHg

    • [E]

      PaO2 of 65 mmHg

  17. A male infant is born via normal spontaneous vaginal delivery at 39 weeks gestation to a 32-year-old G1P1. She had good prenatal care. The infant is heard grunting while trying to breathe, and he shows bilateral subcostal and intercostal retractions. The patient is observed closely for several hours following delivery, and his respiratory status improves over that time. Which of the following is the most likely explanation of this patient’s course?

    • [A]

      Spontaneous reduction of congenital diaphragmatic hernia

    • [B]

      Closure of the ductus arteriosus

    • [C]

      Resorption of excess pulmonary fluid

    • [D]

      Paramyxovirus infection

    • [E]

      Closure of the foramen ovale

  18. A 34-year-old pregnant female at 36 weeks gestation undergoes ultrasound at her obstetrician’s office. The ultrasound reveals polyhydramnios. Which of the following is the LEAST likely diagnosis?

    • [A]

      Duodenal atresia

    • [B]

      Esophageal atresia

    • [C]

      Maternal diabetes

    • [D]

      Fetal posterior urethral valve

    • [E]

      Fetal anencephaly

  19. A 2-week-old infant delivered at 35 weeks gestation is brought to the pediatrician by his mother who reports that he has had a harsh, barking cough and makes a high-pitched whistling sound when he inhales. He has been feeding poorly but has not had a fever. The mother also says that the patient has bouts of blue discoloration around his lips, more frequently when he is lying on his back than on his stomach. Chest x-ray is normal. What is the most likely diagnosis?

    • [A]

      Foreign body aspiration

    • [B]

      Asthma

    • [C]

      Transient tachypnea of the newborn

    • [D]

      Tracheomalacia

    • [E]

      Congenital diaphragmatic hernia

  20. A 1-week-old male presents with intolerance of breast-feedings as well as several episodes of bilious vomiting. On physical exam, the patient’s blood pressure and temperature are normal. The infant appears to be in pain. However, the abdomen does not appear to be distended, and there is no tenderness to palpation. Plain abdominal x-ray shows an absence of gas within the bowel, but is otherwise unremarkable. A complete blood count and electrolytes are normal. What is the next step in the management?

    • [A]

      Admit for observation

    • [B]

      Discharge patient and offer parents reassurance

    • [C]

      Exploratory laparotomy

    • [D]

      Upper GI study with oral contrast

    • [E]

      Abdominal ultrasound

Answers

1. Answer A

Worsening respiratory status with feeding is suggestive of an anatomic or physiologic defect of the upper aerodigestive tract. The next step is to attempt to place an orogastric tube [OGT] and perform AP and lateral chest x-rays. Failure to pass an OGT with radiologic confirmation that the tube is in the upper esophagus is suggestive of esophageal atresia. An olive-shaped mass palpable in the epigastrium [B] suggests a diagnosis of pyloric stenosis. Dyspnea that resolves over the first 24 h of life [C], especially in a full-term neonate who is otherwise healthy, is likely transient tachypnea of the newborn, which is benign and self-limited. Loops of small bowel in the left hemithorax [D] suggest a congenital diaphragmatic hernia, as does a scaphoid abdomen [E] [it indicates that bowel contents are elsewhere such as in the chest].

2. Answer D

Given the history of polyhydramnios, bilious emesis, and Down syndrome, the most likely diagnosis is duodenal atresia. Because of the duodenal obstruction, there would be no gas in the small bowel. Air-fluid levels on abdominal x-ray [A] are characteristic of more distal intestinal obstruction. Inability to pass a nasogastric tube [B] is suggestive of choanal atresia. Abdominal distention with erythema of the overlying skin [C] is concerning for necrotizing enterocolitis. Narrowing of the distal rectum on GI contrast study [E] is found in Hirschsprung’s disease.

3. Answer D

In a premature neonate with rather sudden systemic illness, feeding intolerance, and bloody stools, necrotizing enterocolitis would be the most likely diagnosis. Visible loops of distended bowel and abdominal wall erythema are additional classic findings. Initial treatment is to place the infant NPO, stomach decompression, and administration of IV antibiotics. Surgical management is indicated for suspected perforation, as evidenced by free intraperitoneal air or progressive clinical deterioration [rising WBC count, falling platelet count, worsening acidosis]. Hirschsprung’s disease [A] would present with failure to pass stool at birth and not a sudden decompensation. Duodenal atresia [B] presents with bilious vomiting, but due to the atresia, it presents at birth and with failure to pass meconium. Esophageal atresia with tracheoesophageal fistula [D] would present with respiratory distress during feeding. Meconium ileus [E] classically would present with failure to pass meconium at birth and raises concern for cystic fibrosis.

4. Answer C

Eviscerated bowel without a membrane covering it, with the abdominal wall defect to the right of the umbilicus, is termed gastroschisis. It is more common in young mothers and in those who smoke during pregnancy. Maternal serum alpha-fetoprotein [AFP] tends to be elevated in cases of abdominal wall defects, including both gastroschisis and omphalocele [evisceration though the umbilicus and with a membrane covering bowel]. Typically, maternal serum AFP is greater in gastroschisis than in omphalocele. Maternal serum AFP is checked as part of the triple screen or quad screen that is performed. Elevated maternal serum AFP is seen in other conditions such as multiple gestation, neural tube defects, abruptio placentae, or endodermal sinus tumor, making this a nonspecific marker for abdominal wall defects. Gastroschisis is associated with intestinal atresia. Since the fetus may be unable to swallow amniotic fluid, it is associated with polyhydramnios.

5. Answer D

Gastroesophageal reflux is a common complaint in infants less than 1 year of age. In the first year of age, the pylorus is not fully developed and therefore not fully functional, leading to occasional episodes of reflux [also called spitting up]. Gastroesophageal reflux disease [GERD] is not diagnosed until the infant is failing to gain weight. Treatment of GERD includes thickening the feeds, keeping the infant upright after feeds, and feeding smaller amounts at shorter intervals. Tracheoesophageal fistula typically presents at birth with inability to control secretions. A rare “H” type [fistula without esophageal atresia] often presents in a delayed fashion but would present with recurrent respiratory infections from aspiration and not vomiting [A]. Duodenal atresia would present as bilious vomiting in the newborn period and the classic “double-bubble” sign on abdominal x-ray [B]. Pyloric stenosis presents with non-bilious, projectile vomiting in the first few weeks of life and typically not at 4 months of age [C]. In addition, it would progressively worsen with time. Malrotation would present with abdominal distension and bilious vomiting [E].

6. Answer C

Diffuse pulmonary interstitial and/or alveolar edema suggests transient tachypnea of the newborn, which is self-limited and resolves within 1–2 days. Loops of bowel in the left chest [A] suggest a congenital diaphragmatic hernia. An orogastric tube that does not pass beyond the proximal esophagus [B] suggests an esophageal atresia. Boot-shaped heart with upturned apex [D] is found in infants with tetralogy of Fallot, a constellation of congenital cardiac anomalies, that specifically includes ventricular septal defect, pulmonary stenosis, large overriding aorta, and right ventricular hypertrophy. The double-bubble sign [E] is found in duodenal atresia.

7. Answer A

Bilious vomiting in the neonate should be considered a surgical emergency until proven otherwise. It implies obstruction that is distal to the ampulla of Vater and likely from a congenital anomaly [B–E]. Non-bilious emesis [of milk] is more likely to be physiologic [immaturity of the lower esophageal sphincter]. An exception would be non-bilious vomiting that progressively worsens and is projectile in nature, which would raise suspicion for pyloric stenosis. Since the obstruction is at the pylorus, it does not allow for gastric contents to mix with bile. Pyloric stenosis also presents later, typically in the 3rd week of life.

8. Answer D

The presentation is classic for pyloric stenosis. Although the management of pyloric stenosis is surgical, the first priority in these infants is rehydration. The infant is presenting with signs of moderate to severe dehydration including dry mucous membranes, a sunken fontanelle, delayed capillary refill, and decrease urinary output. The course warrants fluid resuscitation prior to any diagnostic work-up or consultations. After fluid resuscitation, the gold standard imaging modality is ultrasound to assess for pyloric stenosis [E]. Only if ultrasound is negative or equivocal is an upper GI obtained [C]. If imaging modalities demonstrate pyloric stenosis, surgical consultation is then obtained [A]. CT of the abdomen is not warranted in the work-up of suspected pyloric stenosis [B].

9. Answer A

Based on the classic history of polyhydramnios and excessive drooling in a newborn, the patient likely has esophageal atresia. The most important concern is prevention of aspiration. Nutrition [B] can be established after early surgical repair or via a gastrostomy tube if surgery is not undertaken [as in cases where the infant is premature or has pneumonia from aspiration]. Urgent surgical exploration [C] is not indicated, as surgery may be delayed in some patients. Positive pressure ventilation [D] may distend the stomach and cause aspiration. Therefore, it should be avoided when possible. Contrast esophagram [E] is only performed if chest x-ray is nondiagnostic or if the location of the fistula cannot be identified. This study carries risk of aspiration pneumonitis from the contrast agent.

10. Answer E

The first steps in treating a patient with bilious emesis are fluid resuscitation and gastrointestinal decompression [via NG tube]. Once the IV fluid resuscitation has begun, the patient may undergo an upper GI study to evaluate for evidence of midgut volvulus, which may present with distended proximal bowel and a paucity of gas in the distal bowel. Broad-spectrum antibiotics [B] are not indicated as there is no evidence of infection. Operative repair [C] cannot occur until the patient has been stabilized and a diagnosis has been confirmed. Contrast enema [D] would be used to evaluate for Hirschsprung’s disease, which would present as failure to pass meconium and not necessarily bilious emesis.

11. Answer D

This patient has gastroschisis. Since the intestines are outside of the abdominal cavity, insensible fluid losses will be much greater than in an infant without gastroschisis. Therefore, covering of the exposed intestine with moist gauze and IV fluid resuscitation are critical first steps in management. Such patients are also at risk of hypothermia. While exposed intra-abdominal contents increases risk of infection and sepsis [A], this is not as immediate of a concern as is dehydration. After operative repair, patients may be paralyzed to allow the abdominal wall to relax and stretch to accommodate the intestines. If the abdominal cavity is not sufficiently large to accommodate the bowel, the bowel is covered with a silo temporarily. Attempting to forcefully reduce all the small bowels and close it under tension will result in abdominal compartment syndrome, bowel ischemia, and respiratory compromise [B]. Cardiac anomalies [C] are more of a concern in patients with omphalocele than gastroschisis, but regardless this is not an immediate concern. Urinary obstruction [E] is not a typical feature of gastroschisis, though it could rarely occur if the bladder were also herniated through the abdominal wall.

12. Answer A

Laboratory evaluation in a patient with pyloric stenosis classically shows a hypochloric metabolic alkalosis. Chloride is typically lost in the gastric secretions [HCl] via vomiting. Alkalosis is caused by both a loss of protons [HCl] in the gastric fluid as well as secondary to a contraction alkalosis mediated by aldosterone secretion in the setting of hypovolemia. Hypokalemia is a late finding seen in infants who have been vomiting for prolonged period of time, also from contraction alkalosis. The infants also may have a paradoxical aciduria [acidic urine despite alkalosis]. Initially, Na+ in the renal tubule is reabsorbed in exchange for K+ ions but as K+ levels decrease, Na+ is instead exchanged for H+ ions. The presentation of adrenal crisis in an infant may mimic that of pyloric stenosis. However, infants with adrenal crisis typically have hyperkalemic acidosis [D] rather than the hypokalemic alkalosis that is typical of pyloric stenosis [B].

13. Answer E

Ultrasound is the image modality of choice in diagnosing pyloric stenosis, as it does not require any radiation exposure. If ultrasound is negative or equivocal and pyloric stenosis is highly suspected, diagnosis may be attempted with a barium upper GI study with contrast. Upper GI was the test of choice prior to the advent of ultrasound. However, it must be done carefully as it risks causing aspiration given that the infant has a gastric outlet obstruction. Typical findings include elongated pyloric canal [string sign] and delayed gastric emptying. In addition, upper GI studies are further helpful in the setting of a negative ultrasound in order to assess for other items on the differential diagnosis, particularly gastroesophageal reflux. Reassurance would be inappropriate if pyloric stenosis is highly suspected [A]. Operative intervention with pylorotomy would be the definitive treatment should the child be diagnosed with pyloric stenosis, but confirmatory testing should be undertaken first [B]. CT of the abdomen is not an imaging modality of choice for diagnosing pyloric stenosis [C].

14. Answer E

Esophageal anastomotic stricture is a very common long-term complication of esophageal atresia with or without tracheoesophageal fistula repair. These cases may be treated with esophagoscopy with balloon dilation. There is no evidence given to suggest the patient has scleroderma [A], which is particularly unusual to present at age 10. The patient likely does have gastroesophageal reflux [B], as this is another typical side effect of surgical repair, but the symptoms described are more likely attributable to esophageal stricture. Esophageal leak [D] is more commonly a short-term complication, and many leaks will heal spontaneously. Finally, the patient is at increased risk for esophageal cancer [C], but this would be unlikely to develop so early in life.

15. Answer D

This patient likely has necrotizing enterocolitis, which most commonly affects premature infants. It causes necrosis of segments of intestine. In the necrotic segments, gas may be found within the walls of the intestine, a finding known as pneumatosis intestinalis. A palpable olive-shaped mass in the epigastric region [A] suggests pyloric stenosis, which would present with non-bilious emesis, but not severe systemic illness. Double-bubble sign on abdominal x-ray [B] suggests duodenal atresia, which would also not present so acutely. Loops of intestine in the left hemithorax on chest x-ray [C] suggest congenital diaphragmatic hernia, which may present with respiratory difficulty. Donut sign on abdominal ultrasound [E] arises from intussusception of intestine, wherein one segment telescopes into another. This may occur at the ileocecal junction or at the site of a diverticulum and is a surgical emergency. However, it would not necessarily present as acute systemic illness.

16. Answer B

Positive end expiratory pressure [PEEP] is designed to keep alveoli open throughout the respiratory cycle, even during expiration. This increases the surface area available for gas exchange. However, high levels of PEEP will increase the pressure within the alveoli to dangerously high levels and may cause barotrauma, analogous to inflating a balloon until it nearly bursts. Therefore, PEEP should be maintained at or below 5 cm H2O. FiO2 of 100 % [A] will maximize oxygenation. Inhaled NO [C] will cause pulmonary vasodilation, thereby reducing pulmonary hypertension. It has been shown that permissive hypercapnia, with PaCO2 < 60 mmHg [D], and permissive hypoxia, with PaO2 > 60 mmHg [E], allow for lower ventilation settings and less risk of barotrauma.

17. Answer C

The patient described above is an otherwise healthy, full-term newborn whose mother had good prenatal care. Furthermore, his respiratory status improved in the hours after delivery. Therefore, the most likely diagnosis is transient tachypnea of the newborn, which resolves as excess pulmonary fluid is resorbed. Spontaneous reduction of congenital diaphragmatic hernia [A] is very unlikely. Closure of the ductus arteriosus [B] or foramen ovale [E] will have no effect on respiratory status in healthy patients. In patients with some cyanotic heart lesions, such as transposition of the great arteries [wherein the right ventricle ejects into the aorta and the left ventricle ejects into the pulmonary artery], closure of the ductus arteriosus or foramen ovale will have an adverse effect on blood oxygenation. Paramyxovirus [D] can lead to croup, which can lead to respiratory difficulty, but this diagnosis is unlikely given the patient’s presentation.

18. Answer D

Fetal posterior urethral valve prevents the fetus from passing urine via the urogenital tract, which reduces the amount of amniotic fluid present, and is referred to as oligohydramnios. Renal anomalies generally lead to oligohydramnios, whereas intestinal atresias are associated with excess amniotic fluid [polyhydramnios], as the fetus in unable to swallow amniotic fluid. The other items listed above are associated with excess of amniotic fluid.

19. Answer D

Softness of the tracheal cartilage is known as tracheomalacia. Because the cartilaginous support is soft and flexible, the airway can collapse. The condition is usually worse when the patient is supine because gravity pulls the anterior trachea downward toward the posterior trachea, thereby occluding the upper airway. Foreign body aspiration [A] [such as a toy or food particles] would present similarly, but this is unlikely in a 2-week-old. Asthma [B] is also a consideration, but tracheomalacia should be ruled out first. Transient tachypnea of the newborn [C] usually resolves within 1–2 days after delivery. Congenital diaphragmatic hernia [E] generally presents at birth. Delayed presentations rarely do occur, but the chest x-ray would show loops of bowel, or part of the liver, in the thorax.

20. Answer D

Bilious vomiting in an infant should always raise suspicion for midgut volvulus [a complication of malrotation] which left untreated can lead to intestinal gangrene. Yet, early on, ischemic bowel provides few clinical clues to the impending catastrophe, as the physical exam may be benign, with no fever, no abdominal tenderness [as only the visceral peritoneum is initially affected], and normal laboratory values [there are no values that are diagnostic of bowel ischemia]. Similarly, plain abdominal x-ray may be normal. By the time the infant has evidence of systemic inflammation, peritonitis, appears toxic, or has marked leukocytosis, there is a high likelihood that there is gangrenous bowel. The gangrene may involve the entire small bowel. If such a patient survives, he/she may be subject to lifelong intravenous parenteral nutrition or may require small bowel transplantation. Thus further work-up is always required to rule out this potentially devastating problem [A–B]. Given the paucity of findings, it would be premature to take this infant directly to the operating room [C]. Ultrasound [D] is useful for pyloric stenosis [non-bilious vomiting] but not for midgut volvulus. Upper GI study with oral contrast is the best test as it will confirm failure of passage of the contrast, confirming a bowel obstruction, or show a malrotation. A normal study should demonstrate the normal C loop of the duodenum and show that the duodenal jejunal junction is to the left of the spine.

Skin

Questions

  1. A 15-year-old girl develops short gut syndrome following resection of bowel secondary to leiomyosarcoma in the small intestinal wall. She is subsequently placed on long-term total parenteral nutrition [TPN] and is recovering well. A month later, she develops red and inflamed patches of dry and scaly skin around her mouth and eyes. Her hair also begins to thin, and she notices a bad taste when she gets her daily cherry-flavored chloraseptic spray to prevent dry throat. What is the most likely underlying etiology of her skin lesions and thinning hair?

    • [A]

      Zinc deficiency

    • [B]

      Copper deficiency

    • [C]

      Pemphigus vulgaris

    • [D]

      Chromium deficiency

    • [E]

      Psoriasis

  2. A 30-year-old female of Scottish descent presents with a nodule on her face near the corner of her eye. The lesion measures 12 mm in diameter. The borders are irregular, and the center of the lesion is dark. Which of the following is the best recommendation?

    • [A]

      Shave biopsy

    • [B]

      Punch biopsy

    • [C]

      Excisional biopsy with 1 mm margin

    • [D]

      Excisional biopsy with 5 mm margin

    • [E]

      Reexamination in 2 months

  3. A 64-year-old man who emigrated from Japan arrives to his doctor to discuss new skin lesions. His wife first noticed two discolored plaques on his back 2 weeks ago, but he now has multiple lesions all over his back, chest, and face. They are the size of a coin and appear to have a “stuck-on” appearance. He is afebrile, blood pressure is 136/86 mmHg, and he has a pulse of 90/min. The skin lesions do not itch, and they are not tender. He has no other complaints, and a review of systems is negative. He is more concerned about his cosmetic appearance. What is the best next step in management?

    • [A]

      Reexamine in 2 weeks

    • [B]

      Skin biopsy

    • [C]

      Abdominal CT scan

    • [D]

      Mohs procedure

    • [E]

      Corticosteroids

  4. Which of the following would be best suited for Mohs surgery?

    • [A]

      Superficial spreading melanoma in the arm

    • [B]

      Nodular melanoma on the back

    • [C]

      Basal cell carcinoma on the face

    • [D]

      Subungual melanoma

    • [E]

      Squamous cell carcinoma of the neck

  5. A 25-year-old female lifeguard presents to her doctor to discuss a new 10 mm skin lesion that she found on her right forearm that has been growing over the last month. The lesion has a heterogeneous dark blue color, is symmetric, and has been growing vertically. What is the most likely diagnosis?

    • [A]

      Impetigo

    • [B]

      Melanoma

    • [C]

      Nevus

    • [D]

      Molluscum contagiosum

    • [E]

      Squamous cell carcinoma

  6. A 65-year-old obese male with diabetes and a history of IV drug abuse presents with a painful swollen left leg. Exam reveals dark purple discoloration and several large bullae over the calf. Vitals are temperature of 101.1 °F, heart rate of 120/min, and blood pressure of 92/68 mmHg. The CRP is 200 mg/L [normal < 10 mg/L], and the WBC is 28.3/mm3 [normal 4–10/mm3]. Creatinine is 2.0 mg/dL [normal 0.5–1.5 mg/dL], and Na is 127 mEq/L [normal 135–145 mEq/L]. Distal pedal pulses are 1+. IV fluids and intravenous antibiotics are administered. Which of the following is the next best step?

    • [A]

      X-ray of the leg

    • [B]

      CT scan of the leg with IV contrast

    • [C]

      Venous Duplex scan of the left leg

    • [D]

      Measure compartment pressures

    • [E]

      Emergent wide surgical debridement

  7. A 50-year-old field worker arrives to a free clinic to discuss a “sore” on his lower lip. He has had no trauma to the face. He reports that he first noticed the “sore” 6 months ago, and it has slowly gotten bigger. On physical exam, he has an ulcerated 1 cm nodule on his lower lip. There are no telangiectasias present. What is the most likely diagnosis?

    • [A]

      Basal cell carcinoma

    • [B]

      Squamous cell carcinoma

    • [C]

      Lichen planus

    • [D]

      Dermatitis herpetiformis

    • [E]

      Melanoma

  8. Which of the following melanomas have the worst prognosis?

    • [A]

      Superficial spreading

    • [B]

      Nodular

    • [C]

      Lentigo maligna

    • [D]

      Acral lentiginous

    • [E]

      Subungual

  9. Which of the following melanomas do not follow the ABCDE mnemonic?

    • [A]

      Superficial spreading

    • [B]

      Nodular

    • [C]

      Amelanotic

    • [D]

      Acral lentiginous

    • [E]

      Amelanotic and nodular

  10. One day following extensive debridement of the right leg for a necrotizing soft tissue infection [NSTI], a 40-year-old male remains in the ICU, intubated, and requiring 70 % FIO2. White blood cell count has risen from a preoperative level of 16 × 103/μL [normal 4.1–10.9 × 103/μL] to 34 × 103/μL. Serum lactate has also risen. Which of the following is the best next step in treatment?

    • [A]

      Second-look operation

    • [B]

      Amputation of the right leg

    • [C]

      Broaden antibiotic coverage

    • [D]

      CT scan of the leg

    • [E]

      Start pressors

  11. A 45-year-old female presents with a recent change in a preexisting mole on her anterior thigh. She states that the mole keeps bleeding, is darker, and has grown. The mole is 8 mm in diameter on physical exam. There are no palpable nodes in the groin. An excisional biopsy is performed with a 1 mm margin, and to a depth of the subcutaneous fat. Pathology reveals a melanoma that is 0.5 mm in thickness. The margins are negative. What is the next step in the management?

    • [A]

      No further treatment

    • [B]

      Re-excision with 1 cm margins

    • [C]

      Interferon alpha

    • [D]

      Granulocyte-macrophage colony-stimulating factor [GM-CSF]

    • [E]

      Dacarbazine

  12. Which of the following is the most common precancerous skin lesion?

    • [A]

      Actinic keratosis

    • [B]

      Seborrheic dermatitis

    • [C]

      Seborrheic keratosis

    • [D]

      Compound nevi

    • [E]

      Keratoacanthoma

  13. Where are melanomas in patients with dark skin most likely to occur?

    • [A]

      Back

    • [B]

      Arms

    • [C]

      Legs

    • [D]

      Palms, soles, and mucous membrane

    • [E]

      Face

Answers

1. Answer A

Zinc deficiency can occur in surgical patients on long-term total parenteral nutrition or in patients diagnosed with a malabsorption syndrome. This can present with alopecia, red and inflamed patches of dry and scaly skin around the mouth and eyes, abnormal taste, and impaired wound healing. Zinc supplementation will remedy this condition. Copper [B] and chromium [D] deficiency are rare but can also affect this patient population. The most common manifestations of copper deficiency include hematologic abnormalities [anemia, leukopenia] and myeloneuropathy. Chromium deficiency presents with impaired glucose tolerance and peripheral neuropathy. Pemphigus vulgaris [C] occurs as a result of autoimmune destruction of desmosomes between keratinocytes and is characterized by multiple skin and oral mucosa bullae. Psoriasis [E] is believed to have an autoimmune etiology and presents as salmon-colored plaques with a silvery scale that occur on extensor surfaces [e.g., patella].

2. Answer B

The lesion is concerning for melanoma and as such will require tissue confirmation to rule out cancer. Excisional biopsy [removing the entire lesion] [C], down to the subcutaneous fat, would be the preferred approach for a lesion on an extremity or torso. However, depending on the size of the lesion and its location [not desirable to make a cosmetically unappealing large incision if the lesion ends up being benign], an initial incisional biopsy [taking only a small sample] is preferred. Punch biopsy down through the dermis [to calculate Breslow thickness] is the preferred method in this setting. Shave biopsies [A] are not recommended if melanoma is suspected as the true Breslow thickness may not be measurable. During the initial biopsy, no attempts are made to achieve a wide margin. If the pathology comes back benign, no further treatment may be necessary. Excisional biopsy with a 5 mm initial margin [D] would not be indicated as the lesion may be benign. Reexamination [E] is not appropriate for a patient suspected of having melanoma.

3. Answer C

The skin lesions described are most likely to be seborrheic keratosis [SK]. Isolated SKs occur commonly in the elderly. Sudden onset of multiple SKs [Leser-Trelat sign] suggests an underlying carcinoma of the gastrointestinal tract, most often gastric cancer. It is considered to be a result of a paraneoplastic syndrome associated with the cancer. The best next step in working up a suspected GI malignancy is an abdominal CT scan. Given the high likelihood of malignancy, it would be inappropriate to only reexamine the patient in 2 weeks [A]. SKs have a characteristic appearance and typically do not need to be confirmed with a skin biopsy [B]. Mohs [D] is a specialized tissue-sparing procedure for treating skin cancer. It involves tangential excisions of the lesion until margins are negative. Mohs has the advantage in that definitive excision, and closure can be achieved on the same day. Corticosteroids [E] are not used in the management of SKs.

4. Answer C

Mohs is a specialized tissue-sparing technique of treating skin cancer in which the tumor is removed in a series of thin layers as opposed to one wide excision. The advantage is that it prevents excising excessive normal tissue and allows for immediate confirmation of negative surgical margins intraoperatively. It is best suited for basal cell and squamous cell CA in cosmetically sensitive areas such as the face. Mohs is not generally recommended for melanoma. This is because it is difficult to distinguish the normal skin from melanoma on frozen section [immunohistochemical stains are sometimes needed]. Because of this, Mohs is considered by most surgeons to be an unreliable method of resection for melanoma [A, B, D]. The treatment of choice for subungual melanoma is digital amputation.

5. Answer B

Nodular variant melanomas grow vertically, not horizontally. They are usually a uniformly dark blue or black “berry-like” lesion that is mostly symmetric, elevated, and one color. Impetigo [A] is a superficial bacterial infection oftentimes due to Staphylococcus aureus. It presents first as a flat macule and then a raised pustule that erodes and oozes a dry, honey-crusted serum. A nevus [C], or a mole, is described as a small [1 % indicate an intrinsic [renal] etiology of AKI. The most common cause of renal AKI is acute tubular necrosis [ATN]. Hypovolemia [A] causes prerenal azotemia. Prolonged periods of hypovolemia and hypotension lead to poor renal perfusion that directly damage the kidneys and lead to acute tubular necrosis [ATN]. Acute interstitial nephritis [B] also causes intrarenal AKI. However, it is less common than ATN and is an immune-mediated response to certain medications [e.g., penicillin, cephalosporins, sulfa drugs, NSAIDs]. Classic findings include fever, rash, arthralgia, and urinary eosinophilia. Cardiogenic shock [D] can lead to prerenal AKI due to decreased renal perfusion. Prolonged urinary obstruction due to bilateral ureteral obstruction can lead to postrenal AKI. Urine findings are variable. An obstructed Foley [E] is a potential cause of oliguria, but is unlikely to cause AKI and certainly not so soon after surgery.

8. Answer B

The history and exam are most consistent with warfarin-induced skin necrosis. Warfarin inhibits the carboxylation of the vitamin K-dependent clotting factors: II, VII, IX, X, protein C, and protein S. This can acutely lead to the relative deficiency of protein C, owing to its short half-life, and thus can result in an initial hypercoagulable state and subsequent thrombosis in the vasculature supplying the skin. Warfarin-induced skin necrosis is more common in patients who have a preexisting protein C deficiency. Vitamin K deficiency is seen with severe nutritional depletion and intestinal malabsorption and manifests with bruising and hemorrhage [A]. Heparin can cause skin necrosis as well, but this is seen locally at the site of injection [patient however received IV heparin] and in a much smaller distribution [C]. Thrombocytopenia results in petechiae, not skin necrosis [D]. Patients with hemophilia may have a history of deep tissue bleeding into muscles and joints [hemarthrosis] and oftentimes have excessive bleeding after surgical procedures [E], but not skin necrosis.

9. Answer D

The patient has symptoms and signs of a postoperative stroke. Most postoperative strokes are ischemic in nature [not hemorrhagic], and most ischemic strokes are embolic, arising from either the heart [in the setting of atrial fibrillation] or from a plaque at the carotid bifurcation in the neck. This patient, however, has a DVT. The combination of an acute DVT and a stroke suggests a paradoxical embolism, wherein a clot from the venous system enters the systemic [as opposed to pulmonary] circulation. The most likely explanation is an intracardiac shunt such as patent foramen ovale [PFO] or atrial septal defect [ASD]. Such an anomaly would best be demonstrated with an echocardiogram with a bubble study. ECG [A] might be helpful if atrial fibrillation was suspected; however, the patient has a regular rate and rhythm. In the setting described above, a CT of the head would be the first study indicated. Head CT would confirm whether the patient did have a stroke, and whether the stroke was ischemic or hemorrhagic [but this was not an option], but CT would not be helpful for determining the source of an embolic stroke. CT of the chest [B] would be helpful if PE were suspected. Factor V Leiden testing is not routinely recommended following a first-time DVT. Duplex ultrasound of the carotid arteries [E] may identify a plaque, but he is very young to have a carotid stenosis, and the concomitant DVT should raise a higher suspicion for paradoxical embolism.

10. Answer A

In acute tubular necrosis, the renal tubular epithelial cells die and slough off into the urine. These appear as muddy brown casts. Urine osmolarity >500 [B] and bland urine sediment [C] are both consistent with a prerenal AKI state. Red cell casts [D] are suggestive of injury to the glomerulus [e.g., glomerulonephritis]. White cell casts [E] are suggestive of tubulointerstitial disease or acute pyelonephritis but may also be observed with many glomerular disorders.

11. Answer D

The patient is homeless, which predisposes him to unsanitary conditions. Poorly controlled diabetes itself is an immunosuppressed state. Given this information about the patient, the presence of leukocytosis with neutrophilic predominance, and his physical exam findings [e.g., painful, erythematous, swollen leg with bullae and violaceous skin], this patient likely has necrotizing fasciitis. Management consists of blood cultures, broad-spectrum antibiotics, and urgent surgical debridement. Antibiotics and cultures alone would not be appropriate [E]. Choices A–C are all appropriate considerations for patients presenting with a PE secondary to DVT.

12. Answer D

The patient is in shock. Given the high cardiac output, and low systemic vascular resistance, septic and anaphylactic shock are the most likely. However, since the patient has not had any medications or unusual exposures and is breathing normally, it is most likely septic shock. Patients with difficult urinary catheterizations may have subsequent bacteremia which can result in septic shock. The first step in management of septic shock is aggressive IV fluid resuscitation with either normal saline [NS] or lactated Ringer’s [LR]. Norepinephrine [E] is considered as the first-line vasopressor for septic shock. Additionally, vasopressin can be used in combination with norepinephrine. Epinephrine is also used for septic shock, but after the above two. Dopamine [B] was initially believed to increase renal perfusion in patients with shock, but studies have failed to consistently demonstrate this. It is not typically recommended for patients with septic shock [except for the rare patient with associated bradycardia]. Phenylephrine [A] is not recommended for septic shock except in highly selected patients. All patients with septic shock should also receive blood cultures before starting broad-spectrum antibiotics [C].

13. Answer B

In a young patient presenting with recurrent epistaxis, isolated thrombocytopenia, and bleeding symptoms, an isolated acquired thrombocytopenia should be considered. Immune thrombocytopenic purpura [ITP] is an autoimmune disease characterized by autoantibodies against platelets and thus is considered a consumptive process. In a patient with a platelet count

Bài Viết Liên Quan

Chủ Đề